You are on page 1of 108

STEPS TO SUCCESS

This section presents an effective, step-by-step approach to use when answering questions on
Part 5 of the TOEIC'~test.

The steps shown here are designed to help you achieve a higher score when you take the test.
By following these steps, you should be able to maximize your score on this part of the test.

a Quickly read the sentence and look at the answer choices. Identify the language focus
and ask yourself what exactly the item is testing.

a Read the sentence again carefully. First, think about the meaning of the sentence as a
whole. Then focus on the missing word or phrase .
• Ask yourself what kind of word you need to complete the sentence (a noun, verb,
adjective, etc.) .
• Pay close attention to the words that come before and after the blank. These words
often provide context clues that will help you identify the correct answer choice.

Grammar often tested ... Vocabulary often tested ...


word choice word forms
auxiliary verbs words with similar meanings
comparatives and superlatives word choice
subject-verb agreement prepositions
pronouns words that look alike
verb forms and tenses conjunctions
gerunds and infinitives phrasal verbs

II Look at the answer choices and identify the part of speech and meaning of each one.
Eliminate any you are sure are incorrect. If you can't decide between two or more
options, read the sentence silently to yourself with each answer choice in place.
Which one "sounds" right?

II Mark your answer.

PART 5 INCOMPLETE SENTENCES 193


PART 5 STEPS TO SUCCESS Practice 1
1.1
~ Quickly read this sentence and the answer choices. What exactly is the item testing?

The baseball coach appeared to be than the players before the game.
(A) nervous
(B)' more nervous
(C) nervously
(D) most nervous

] word choice [ ] subject-verb agreement


] gerunds and infinitives [,r] comparatives and superlatives

EmD Read the sentence again carefully. Think about the form of the word or phrase you need.
Study the words that come before and after the blank.

mmD Go through each of the answer choices and eliminate any you are sure are incorrect. If you
are unsure, ask yourself which answer choice "sounds" right.

I!DD Mark your answer.


@@@@

1.2 I
~ Quickly read this sentence and the answer choices. What exactly is the item testing?

......................
the bad weather forecast, it was agreed that the staff picnic would go ahead as
scheduled.
I
(A) However
(B) Although
(C) Because
(D) Despite

] prepositions ] word forms


] conjunctions I phrasal verbs

EmD Read the sentence again carefully. Think about the meaning of the word you need. Study
the words that come immediately after the blank.

mmD Go through each of the answer choices and eliminate any you are sure are incorrect. If you
are unsure, ask yourself which answer choice "sounds" right.
I

I!DD Mark your answer.


@@@@
I
I

194 READING
PART 5 STEPS TO SUCCESS
Practice 2
1.1
~ Quickly read this sentence and the answer choices. What exactly is the item testing?

I thanked Daniel because I . not have completed the report without his help.
(A) must
(8) should
(C) could
(D) had

1 word choice 1 subject-verb agreement


1 auxiliary verbs 1 pronouns

mmD Read the sentence again carefully. Think about the meaning of the word you need. Study
the words that come before and after the blank.

mmD Go through each of the answer choices and eliminate any you are sure are incorrect. If you
are unsure, ask yourself which answer choice "sounds" right.

l!DD Mark your answer.


0@@@

1.2
~Quickly read this sentence and the answer choices. What exactly is the item testing?

This is the first customer complaint about poor service that I have ...........
in two years.
(A) worked out
(8) called off
(C) cut down
(D) come across

1 prepositions 1 word forms


1 phrasal verbs 1 words with similar meanings

mmD Read the sentence again carefully. Think about the meaning of the phrase you need. Study
the words that come before and after the blank.

mmD Go through each of the answer choices and eliminate any you are sure are incorrect. If you
are unsure, ask yourself which answer choice "sounds" right.

l!DD Mark your answer.


0@@@

PART 5 INCOMPLETE SENTENCES 195


PART 5 STEPS TO SUCCESS Practice 3
1.1
EmD Quickly read this sentence and the answer choices. What exactly is the item testing?

There was nobody to help me copy the files, so I had to do it ..


(A) myself
(B) mine
(C) me
(D) my

1 verb forms and tenses I subject-verb agreement


1 auxiliary verbs 1 pronouns

ED Read the sentence again carefully. Think about the part of speech of the word you need.
Study the words that come before the blank.

mmD Go through each of the answer choices and eliminate any you are sure are incorrect. If you
are unsure, ask yourself which answer choice "sounds" right.

I!D!D Mark your answer.

0@@@

1.2
EmD Quickly read this sentence and the answer choices. What exactly is the item testing?

This report is very of the health and safety standards at the factory.
(A) critic
(B) critical
(C) criticize
(D) criticism

1 word choice 1 word forms


1 words with similar meanings 1 phrasal verbs

ED Read the sentence again carefully. Think about the part of speech of the word you need.
Study the words that come before and after the blank.

mmD Go through each of the answer choices and eliminate any you are sure are incorrect. If you
are unsure, ask yourself which answer choice "sounds" right.

I!D!D Mark your answer.

0@@@

196 READING
PART 5 STEPS TO SUCCESS
Practice 4
1.1
ED Quickly read this sentence and the answer choices. What exactly is the item testing?

We regret to inform you that on this occasion you have been . ......in your application
for the position of Assistant Shift Manager.
(A) unsuccessful
(B) ineffective
(C) incompetent
(0) unproductive

] word choice 1 prepositions


] phrasal verbs 1 conjunctions

ED Read the sentence again carefully. Think about the meaning of the word you need. Study
the words that come before and after the blank.

mm Go through each of the answer choices and eliminate any you are sure are incorrect. If you
are unsure, ask yourself which answer choice "sounds" right.

ED Mark your answer.


@@@@

1.2
mmD Quickly read this sentence and the answer choices. What exactly is the item testing?
Because our favorite restaurant , we decided to go home and cook instead.
(Al to close
(B) closes
(C) be closed
(D) was closed

1 auxiliary verbs 1 subject-verb agreement


] gerunds and infinitives 1 verb forms and tenses

ED Read the sentence again carefully. Think about the tense of the word or phrase you need.
Study the words that come before and after the blank.

mm Go through each of the answer choices and eliminate any you are sure are incorrect. If you
are unsure, ask yourself which answer choice "sounds" right.

ED Mark your answer.


@@@@

PART 5 INCOMPLETE SENTENCES 197


PART 5 STEPS TO SUCCESS Practice 5
1.1
•• Q~ickly read this sentence and the answer choices. What exactly is the item testing?
I
Can you please stop.... that and help me with this?
(A) to do
(B) doing
(C) you do
(D) do

1 verb forms and tenses 1 auxiliary verbs


1 pronouns 1 gerunds and infinitives
I
I!DD R~ad the sentence again carefully. Think about the form of the word or phrase you need.
Study the words that come before and after the blank.

Iii!D Go through each of the answer choices and eliminate any you are sure are incorrect. If you
are unsure, ask yourself which answer choice "sounds" right.

I!iiiD Mark your answer.

0@@@

1.2
•• Quickly read this sentence and the answer choices. What exactly is the item testing?

For the whole of next week, Mrs. Philipps will be a trade convention in Frankfurt.
(A) out
(B)to
(C) at
(D) across

1 words with similar meanings 1 word choice


1 prepositions 1 conjunctions
I
I!DD Read the sentence again carefully. Think about the meaning of the word you need. Study
the words that come before and after the blank.

Iii!D Go through each of the answer choices and eliminate any you are sure are incorrect. If you
are unsure, ask yourself which answer choice "sounds" right.

I!iiiD Mark
I
your answer.

0@@@

198 READING
Strategy Review and Tips

Strategy Review Reading Test - Part 5

Remember, in the test...

Read the whole sentence carefully. Do not stop when you see the blank. This will help you to get an
idea of the context.

Decide if the focus is grammar or vocabulary. Then look more closely and ask yourself what exactly
is being tested.

Look at the words before and after the blank and try to identify the part of speech, and the meaning,
of each missing word or phrase.

Eliminate any answers that you are sure are wrong.

If you are not sure about the answer, don't waste time. Decide quickly and move on to the next item.

= Here is some advice that people taking the TOEIC test have found usefuf for this part.

----------~-----------
Choose the tips you like. and try to use them .
._-- ----

.0

• • •• •

••

• 0
••

PART 5 INCOMPLETE SENTENCES 199


Revie~ Test
I
Directions: Read each sentence. You will notice that there is a word or phrase missing. Study the four
answer choices and select the one answer: (A), (8), (C), or (D), that best completes the sentence.
Then mark your answer.

1. None of the we interviewed for this 6. May I please speak to the person for
job was suitable. maintaining the website.
(A) applies (A) response
(8) applications (8) responsible
(C) applicators (C) responsibility
(D) appliJants (D) responsive

2. More people would be able to use our products 7. Luckily, Derek nor Alicia were hurt in
without difficulty if the instructions were the accident at the factory.
(A) either
(A) simplify (8) both
(8) simplicity (C) neither
(C) simplified (D) but
(D) simply
8. Can everyone please help to tea and
3. Senior experts seem to think that the economic coffee while we wait for the speaker to arrive?
................................ 1. will begin early next year. (A) they
I
(A) recovery (8) them
(8) recover (C) theirs
(C) recovered (D) themselves
(D) recovering
9. If we want to maintain the same rate of
l
4. The room was so I could not find a ..................
next year, we will have to invest in
seat and had to stand up for the lecture. new machinery and equipment.
(A) crowded
,
(A) contribution
(8) isolated (8) routine
(C) complete (C) event
(D) incapable (D) growth
I
5. I would like to introduce you to my personal 10. The board decided to approve our plan
.......................
, Ms. Ono. even though it was option under
(A) assist consideration.
(8) assistance (A) more expensive
. I
(C) assistant (8) the most expensive
(D) assisting (C) more expensive than
(D) as expensive

1.@ @ @ @ 4. @ @ @ @ 7. @ @ @ @ 10. @ @ @ @
2.@ @ @ @ 5. @ @ @ @ 8. @ @ @ @
3.@ @ @ @ 6. @ @ @ @ 9. @ @ @ @

200 READING
11. From the of the robber, he was tall 17. The main of this plan is that the
with brown hair and a moustache. company should be able to increase profits
(A) reception while at the same time minimize its expenses.
(B) prescription (A) method
(C) description (B) advantage
(D) conception (C) image
(D) boost
12. Making large cuts in the budget will not be
..................
to achieve, but it is necessary. 18. Astronomers have a new planet
(A) easy outside our solar system.
(B) easily (A) discovered
(C) ease (B) invented
(D) eased (C) created
(D) constructed
13. The seminar room we are using today is
............................
the third floor, across from the 19 they worked hard all day, by the
elevator. evening they still had not finished their tasks.
(A) at (A) Aithough
(B) over (B) However
(C) on (C) Despite
(D) by (D) Even

14. I do not want to hold you in case you 20. I am taking a special course because I
miss your train, so go ahead and we can talk ..........................
to improve my presentation skills.
about this next Monday. (A) am wanting
(A) up (B) will want
(B) in (C) would want
(C) out (D) want
(D) over
21. There is a lot of work to do, so let's try to
15. At this time of uncertainty and crisis, the most ............................
as much progress as we can as
...............
thing we could do is to rush into a quickly as possible .
hasty decision. (A) do
(A) irresponsible (B) become
(B) irritable (C) make
(C) irresistible (D) obtain
(D) irreversible
22. We the same vendor to provide all our
16. We were very excited by the.... news food services since 2010.
that Joseph and Francesca were engaged. (A) have used
(A) wonder (B) were using
(B) wonderful (e) had used
(C) wondering (D) used
(D) wondered

11.@ @ @ @ 14. @ @ @ @ 17. @ @ @ @ 20. @ @ @ @


12.@ @ @ @ 15. @ @ @ @ 18. @ @ @ @ 21. @ @ @ @
13.@ @ @ @ 16. @ @ @ @ 19. @ @ @ @ 22. @ @ @ @

PART 5 INCOMPLETE SENTENCES 201


23. I will meet you for coffee ten minutes 27. I do not think the way Economics .
in the cafeteria. taught in college prepares anyone for a career
(A) until in business.
(B) during (A) are
(C) in (B) be
(D) before (C) is
(D) has
24. In case there is a power outage, an
emergency generator in the basement will 28. Do you think you help me redecorate
supply power to the hospital. the bathroom next weekend?
(A) tempting (A) would
(B) tentative (B) must
(C) temporary (C) may
(D) terminal (D) could

25. The invoice from the for the building 29. I found a $20 . . on the ground in the
extension was far more than we had expected. parking lot when I arrived at work this morning.
(A) contractor (A) coin
(B) conductor (B) cash
(C) computer (C) money
(D) consumer (D) bill

26. We will not know the training program 30. Serina was so busy compieting the schedules
has been effective until next year. that she did not have time to have
(A) because lunch.
(B) not only (A) many
(C) whether (B) any
(D) neither (C) some
(D) no

23.(£; @ @ @ 26. (£; @ @ @ 29. (£; @ @ @


24.(£; @ @ @ 27. (£; @ @ @ 30. (£; @ @ @
25.(£; @ @ @ 28. (£; @ @ @ Score ... . ./30

r
ExTRA PRACTICE ONLINE !
~
, , ~.pass.th~-toeic.test.com fo~~ore ~;ammar and vocabulary exercises to help
improve your score on the TOEle test.
i
L. ._ .!-__________ - _
202 READING
PART
6 TEXT COMPLETION

This part of the TOEIC~ test consists of four short texts. Each text has four blanks -
these are spaces where a word, phrase, or sentence is missing. Below each blank are
four answer choices. You must choose the answer that best completes the sentence.

Part 6 of the TOEIC test is very similar to Part 5. The main focus is on grammar and
vocabulary. However, Part 5 consists of single sentences whereas in Part 6 the sentences
are part of a text (an e-mail, article, letter, notice, etc.). Therefore, you sometimes need to
read more than one sentence in order to choose the correct answer.

In addition, one question for each text asks you to identify a missing sentence. This
question tests your understanding of context. You may need to read the paragraph, or
even the whole text, again to get a general idea of the purpose.

QUICK CHECK

H
How many questions are in this lJart of the TOEIC test?
H
How many answer choices are there for each question?

W
H
How

W ....
is Part 6 different from Part 5?
•.

You may find it useful to review the Grammar and Vocabuiary sections before you start.

PART 6 TEXT COMPLETION 203


~~TRYITOUT

EXAMPLE

SENTINEL SYSTEMS

The safety of our employees is a top priority at Sentinel Systems. Every member of the
production staff is therefore required 1 the "Safety in the Workplace" training
seminar once every year. This is a mandatory requirement to make sure all employees work
in a safe environment. Anyone who? to attend the seminar risks endangering
themselves and their coworkers.

The seminars take place on the last Friday of every month, at 2:00 P.M. in Training Room B.
A maximum of twenty people can attend each session. Please note that we :3_ .
have four spaces left for this month's seminar on Friday the 27th. Call Amanda at Ext. 233
to reserve your spot.

4 If you have any questions, contact me bye-mail.


Geoff Kleber
Human Resources

. ..
1. (A) attendance
(B) attending
(C) to attend
~~. -

(0) attend

2. (A) wishes
(B) registers
(C) applies
(0) fails

3. (A) currently
(B) openly
(C) absolutely
(0) hardly

4 (A) I look forward to seeing you there.


(B) Thank you for your co-operation.
(C) No further action is necessary.
(0) Your concern is most appreciated.

204 READING
Directions: Read each text. You will notice that there are four blanks. These are places where a word,
phrase, or sentence is missing. For each blank, study the four answer choices and select the one
answer: (A). (B). (C), or (0). that best compietes the text. Then mark your answer.

Questions 1-4 refer to the following e-mail.

From: Sandie.Jenkins@kkinc.com
To: Undisclosed Recipients
Re: New website

I am pleased to inform you all that the new KK, Inc. website is finally operational. This is great news.
After months and months of hard work, the resuit is very impressive.

1 As members of our Customer Review Panel, your contributions have been extremely
useful. ? your help, I don't think we could have succeeded in creating such a good
website.

As a token of our appreciation for your efforts, we ~ like to give each of you a $200 gift
certificate. Please log on to the website and enter the code JYIU987R to claim your reward.
We hope you have fun L1 this in our online store!

Sincerely,
Sandie Jenkins
Customer Liaison Officer, KK, Inc.

1. (A) Thank you for all your suggestions. 3. (A) could


(B) Good luck in your future endeavors. (B) might
(C) Please send in your responses soon. (C) would
(0) Be sure to claim your reward. (0) will

2. (A) Because of 4. (A) spending


(B) Without (B) spend
(C) Apart from (C) to spend
(0) Not only (0) spent

1.0 @ @ @ 2. 0 @ @ @ 3. 0 @ @ @ 4. 0 @ @ @

PART 6 TEXT COMPLETION 205


Questions 5-8 refer to the following article.

A group of international business experts today 5 the results of a three-year economic


survey. Their research indicates that a global recovery might be on the way. ..... 6 The
group, called Business International, made the statement in a press conference at the Governor's
Hotel, New York. Over fifty industry economists participated in the data review. After the forty-five
minute presentation, there was a question and answer session. "We are confident that economic
growth will occur 7 the next two years," said Dr. Neil Ormond, the president of
Business International. ".... 8 ....not all sectors of the world economy will see improvements,
the overall effect should be positive."

5. (A) pronounced 7. (A) on


(B) announced (B) to
(e) denounced (C) in
(D) renounced (D) at

6. (A) Markets worldwide welcomed the news. 8. (A) Although


(B) Further economic data is needed. (B) However
(C) This report will be completed soon. (C) Despite
(D) Nobody has taken responsibility yet. (D) Even

5.(£) @ @ @ 6. (£) @ @ @ 7. (£) @ @ @ 8. (£) @ @ @

Score ... .../8

206 READING
STEPS TO SUCCESS
This section presents an effective, step-by-step approach to use when answering questions on
Part 6 of the TOEIC" test.

The steps shown here are designed to help you achieve a higher score when you take the test.
By following these steps, you should be abie to maximize your score on this part of the test.

II. Preview the text quickly to get a general idea. Do not look at the answer choices.

Find the first biank. Is this testing grammar/vocabulary, or is it a missing sentence that
tests reading comprehension?
• For a grammar/vocabulary question, ask yourself what kind of word or phrase you
need to complete the sentence (a modal verb, noun, etc.). Study the words that
come before and after the blank space .
• For a missing sentence, ask yourself what the purpose of the text is and look at how
the missing sentence connects with the sentences that come before and after.

II Remember that sometimes you may need to refer to the previous and following
sentences to help you find the correct answer. Occasionally you will need to read
the whole text again.

II Study the answer choices carefully, and eliminate any you are sure are incorrect.
Mark your answer. Then repeat steps 2 through 4 for the remaining questions.

~;-":---~'- " . ---


.'c.
. ..•.
Previewing
-
KeySkill for Part 6
:,::Step 1 involves previewing. Here, that means reading the text quickly. Do not worry about the
....missing words or read the answer choices. Do not read every word - just focus on the key words
J: that contain important information. Your goal is to identify the type of text (a letter, e-mail, etc.), .~
i.and to get an overall understanding of what it is about. Get used to previewing Part 6 texts in this : I
I
r' way, and you will improve your TOEIC score.
~';.2...::..:...._.__ .__ ... _'.. __ . - ;. _. .. _, ~ .__
.j
.:........:.<~

PART 6 TEXT COMPLETION 207


PART 6 STEPS TO SUCCESS Practice 1
EmD Quickly preview this advertisement.

Fantastic Summer Sale!


The HomeTronics summer sale starts Monday. Don't miss the fantastic summer ~ .
on a huge range of electrical products. Take 50% off all flat screen televisions, MP3 players,
and satellite receivers. Plus, for the next two weeks, we are offering shoppers the chance to
enter our free prize drawing. For every purchase you 2 , we will give you a ticket
to win a two ..week dream vacation.. 3 Europe plus $1 ,000 in spending money .
.... ~ Hurry in now to the HomeTronics summer sale. At these prices, you'd be crazy
to shop anywhere else!

1. (A) exclusions 3. (A) over


(B) withdrawals (B) on
(C) discounts (C) in
(D) cutbacks (D) at

2. (A) make 4. (A) All items have been reduced.


(B) will make (B) We hope to see you there.
(C) are making (C) No exchange is allowed.
(D) made (D) What are you waiting for?

•• Read the first sentence with a blank, and look at the answer choices. Is the focus grammar
or vocabulary? What exactly is the item testing?

•• Read the sentence again. Study the words that come before and after the blank. If
necessary, look for information in previous and following sentences. For a missing
sentence, you may need to read the whole text again.

I!D!D Eliminate any answer choices you are sure are incorrect. Mark your answer. Then repeat
steps 2 through 4 for the remaining questions.

1.0 @ @ @

2.0 @ @ @

3.0 @ @ @

4.0 @ @ @

208 READING
PART 6 STEPS TO SUCCESS
Practice 2
&mD Quickly preview this notice.

Staff Parking

I'd like to J all staff that it is not possible to park in the main parking lot at the
front of the building. As you know, these spaces are limited, and are strictly for the use of our
customers and other visitors. Sufficient staff parking 2 at the rear of the building.
Staff parked outside the main entrance will be asked to move. 3 ...cars immediately.

Some customers have complained that they are unable to find spaces. 4 Thank
you for your attention.

Carole Wynn, Manager

1. (A) remind 3. (A) they


(8) remember (8) theirs
(C) repeat (C) their
(0) reclaim (0) them

2. (A) provides 4. (A) Comments are always welcome.


(8) to provide (8) This situation has to stop.
(C) providing (C) Your feedback is appreciated.
(0) is provided (0) There is space for everyone.

BiD Read the first sentence with a blank, and look at the answer choices. Is the focus grammar
or vocabulary? What exactly is the item testing?

IlimD Read the sentence again. Study the words that come before and after the blank. If
necessary, look for information in previous and following sentences. For a missing
sentence, you may need to read the whole text again

EiiiD Eliminate any answer choices you are sure are incorrect. Mark your answer. Then repeat
steps 2 through 4 for the remaining questions.

1.0 @ @ @
2.0 @ @ @
3.0 @ @ @
4.0 @ @ @

PART 6 TEXT COMPLETION 209


PART 6 STEPS TO SUCCESS Practice 3
I'!lED Quickly preview this article.

Business Expert Visits City

.................
1 There is huge excitement at the prospect of meeting the boss of one of the
nation's largest and most successful enterprises. Ms. Jackson, the millionaire owner of the
Happy Chicken chain of restaurants, is here to offer help to the region's small businesses. She
is giving a talk to business owners Thursday at the Maryland Convention Center.
...............
?: the vital role they play in our economy, many companies lack the support they
need to realize their growth potential," she said. "My talks give entrepreneurs the right advice
to help them ~...... .. their goals." Ms. Jackson recently appeared in the hit TV show
"Back to Business," and will host a free networking event at the Maryland Convention Center
.......... '.1. .. June 4. For more details, visit www.thebiz.com/events.

1. (A) It helps to be a celebrity. 3. (A) win


(B) More firms are reporting growth. (B) achieve
(C) Louise Jackson is in town. (C) gain
(D) Female bosses are popular. (D) succeed

2. (A) Although 4. (A) on


(B) However (B) at
(C) Whereas (C) in
(D) Despite (D) by

I!ii!D Read the first sentence with a blank, and look at the answer choices. Is the focus grammar
or vocabulary? What exactly is the item testing?

mmD Read the sentence again. Study the words that come before and after the blank. If
necessary, look for information in previous and following sentences. For a missing
sentence, you may need to read the whole text again.

Eii!D Eliminate any answer choices you are sure are incorrect. Mark your answer. Then repeat
steps 2 through 4 for the remaining questions.

0
1. @ @ @

2.0 @ @ @

3.0 @ @ @

4.0 @ @ @

210 READING
PART 6 STEPS TO SUCCESS
Practice 4
I!iiiD Quickly preview this letter.

Mr. Frank Leibnitz


8974 Green Valley Drive
Inglewood, CA 90305 June 17

Dear Mr. Leibnitz:

Thank you for attending the interview with us on June 3 for the Healthcare Assistant vacancy
at Dews Retirement Homes.

Although your qualifications are impressive, ..1 it is clear you have a great deal of
useful experience, I regret that we are not able to offer you a.......... 2 at this time.

There were many strong candidates, and unfortunately, on this occasion you have not
been successful.

..............;J. I wish you success with your 4 for suitable employment.

Yours sincerely,
Raymond Lee

1. (A) nor 3. (A) All employees are carefully selected.


(8) and (8) Thank you for your interest in our company.
(C) but (C) Your participation was welcome.
(D) then (D) We were delighted to meet you.

2. (A) condition 4. (A) research


(8) situation (8) searching
(C) position (C) searcher
(D) preparation (D) search

IlmD Read the first sentence with a blank, and look at the answer choices. Is the focus grammar
or vocabulary? What exactly is the item testing?

mmD Read the sentence again. Study the words that come before and after the blank. If
necessary, look for information in previous and following sentences. For a missing
sentence, you may need to read the whole text again.

I!imD Eliminate any answer choices you are sure are incorrect. Mark your answer. Then repeat
steps 2 through 4 for the remaining questions.

1.@ @ @ @
2.@ @ @ @

3.@ @ @ @
4.@ @ @ @

PART 6 TEXT COMPLETION 211


PART 6 STEPS TO SUCCESS
Practice 5
•• Quickly preview this memo.

From: Dayna.Black@brm.com
To: "All Employees" <allstaff@brm.com>
Re: Employee of the Season

Yes, it is that time J ! Voting forms will be issued today. As done previously, the
forms will be placed in envelopes and distributed to you by your line managers at your
next departmental meetings. The forms must be completed and 2 to your line
manager by no later than midday on Friday, March 25th.
You can vote for your 1st, 2nd, and 3rd choices. People receiving votes from outside of their
own department will receive double points. Please remember, you must provide a reason to
support your ~
We will announce the winners at the next staff meeting planned for Wednesday, March 30th.
...... .....<1........ If anyone has any questions, please let me know.

Dayna
Staff Development

1. (A) yet 3. (A) choice


(B) already (B) cause
(C) still (C) business
(D) just (D) claim

2. (A) returned 4. (A) Membership is not mandatory.


(B) revisited (B) Can all staff report on time?
(C) replied (C) No exceptions can be made.
(D) replaced (D) Please do try to attend.

IiiiiD Read the first sentence with a blank, and look at the answer choices. Is the focus grammar
or vocabulary? What exactly is the item testing?

Em» Read the sentence again. Study the words that come before and after the blank. If
necessary, look for information in previous and following sentences. For a missing
sentence, you may need to read the whole text again.

mmD Eliminate any answer choices you are sure are incorrect. Mark your answer. Then repeat
steps 2 through 4 for the remaining questions.

1. @ @ @ @
2.@ @ @ @

3.@ @ @ @

4.@ @ @ @

212 READING
Strategy Review and Tips

Strategy Review Reading Test - Part 6


Remember, in the test. ..

Begin by previewing the passage. Don't stop when you see a blank. Keep reading. You should read
quickly to get a general idea of the content.

Carefully read each sentence with a blank. Look closely and decide what is being tested: grammar,
vocabulary, or (for a missing sentence) reading comprehension.

Look at the words before and after the blank and try to identify the part of speech, and the meaning,
of each missing word or phrase.

Eliminate any answers that you are sure are wrong.

Do not forget that sometimes you may need to look at other sentences elsewhere in the text to help
you identify the correct answer - especially for missing sentences.

If you are not sure about the answer, don't waste time. Decide quickly and move on to the next item.

am Here is some advice that people taking the TOEIC test have found useful for this part.
Choose the tips you like, and try to use them.

.'
"I try to predict the missing words before I look at the answer choices. Then, when
J check the answer choices, if I find the same word or something similar I know it is
probably the correct answer." Kumiko Ito
'~==~-,-_ ..-c', --~-_. __ -----~~~~~~~~~-_~--""

. . _. ----- -_.- -
. "In Part 6, I never read the text. I go straight to the ~i~sing words. I can usually _ . 1
.~o_:n::w:.r~ ~~atdefinit~_~ d_o_n'_t~ .j!

" -

"For missing sentence questions, ask yourself what is the purpose of the text." 1
, Ana Velasquez »
'--'A':"-:"=.~':""~-"""-_=.

, ----
"If the information you need is not in the sentence with a blank, always start by looking
at the sentence immediately before the sentence with the blank. You'll often find what
you need there:' Haruo Matsumoto
.'"'-- -=-~---==_.
__ ..._-

. -
,
"Don't spend long on this part. There are only 16 points. Do it as quickly as you can I

and move straight on to Part 7." Soon-Bok Yoon


.)

PART 6 TEXT COMPLETION 213


Review Test
Directions: Read each text. You will notice that there are four blanks. These are places where a word,
phrase, or sentence is missing. For each blank, study the four answer choices and select the one
answer: (A), (8), (C), or (0), that best completes the text. Then mark your answer.

Questions 1-4 refer to the following article.

Mishima Motors to Expand


By Hideo Arakawa,Japan

Mishima Motors today announced plans to . 1 expand its car manufacturing operations.

Sales of the 'Oaiba' - the company's most popular model - are partially responsible for this move.
"The 'Oaiba' ~ great success since its launch," said C.E.O Minoru Takada. "We now
want to double our production over the next three years, by starting production at new facilities in
Tokyo, Osaka, and Sapporo." The company's stock price rose an incredible 21 percent on hearing
this news, ~ fears by some experts that Mishima Motors is being too aggressive.

Mr. Takada welcomed the jump in the stock price. "With such a positive response
t o our am b't'
I tOUS program, we canna t fai,'1" he Insls
. . t ed 4 .

1. (A) significantly 3. (Al even though


(8) fluently (8) in spite of
(C) strictly (C) however
(0) commonly (0) so that

2. (A) will enjoy 4. (A) Stock prices can go down, too!


(8) was enjoying (8) A new program will start soon.
(C) has enjoyed (C) We appreciate your order.
(0) had enjoyed (0) The time is right for expansion.

1.(£)@@@ 2. (£) @ @ @ 3. (£) @ @ @ 4. (£) @ @ @

214 READING
Questions 5-8 refer to the following letter.

Mrs. Maggie Bertram


1025 Francis Street
Vancouver
British Columbia
Canada V6Z 1L4

August 14

Dear Mrs. Bertram:

Our records show that you last 5 .. us on February 26, earlier this year. Your next dental
appointment is now due. Please call 360-925-8144 to make an appointment as soon as possible.

You may be interested to know that we are offering our customers 15% off our teeth
whitening ~ The most important thing you wear is your smile, so ask about our Laser
Plus package, and how it can get you ahead. Call us today to take 7 of this great offer.
8

Patient Support
Bannard Associates

5. (A) visit 7. (A) advice


(B) visited (B) notice
(C) have visited (C) advantage
(D) were visiting (D) possession

6. (A) renewal 8. (A) Get the smile you deserve!


(B) provision (B) Come and join us.
(C) resource (C) Your views are most appreciated.
(D) service (D) Experience is not essential.

5.0 @ @ @ 6. 0 @ @ @ 7. 0 @ @ @ 8. 0 @ @ @

PART 6 TEXT COMPLETION 215


Questions 9-12 refer to the following e-mail.

To: jgooding@FastNet.com
From: sales@mustick.com
Subject: Your mustick.com order

Dear Mr. Gooding:

Thank you for ordering four e-tickets to see The Big Bash Live on Saturday, April 3 at the M2
Arena... !!. Your application for tickets..... ...1..~ being processed.

You ~..~ receive your tickets bye-mail within two weeks. Simply print them, and take
them with you to the event. You will need to show them to access the stadium. If you do not
hear from us within two weeks, please .... ~? the Ticket Services team bye-mailing ticket-
services@mustick.com.

Thank you for choosing mustick.com.

Yours sincerely,

Jason Chen
Ticket Services

9. (A) We are glad you enjoyed the show. 11. (A) could
(B) The ticket price cannot be changed. (B) should
(C) We have received your request. (C) must
(D) Tickets to this event are very popular. (D) have to

10. (A) is 12. (A) to contact


(B) are (B) contacting
(C) will (C) contacted
(D) has (D) contact

9. (£) @ @ @ 10. (£) @ @ @ 11. (£) @ @ @ 12. (£) @ @ @

216 READING
Questions 13-16 refer to the following notice.

Watch out for phone scams


...........
~.~ someone ever call saying you owed tax and demanding you make a payment
immediately? Did you ever get a phone call 1..~ you of a refund, and asking for your
banking details and proof of identity? If the answer is affirmative, then that was a scam!

Phone scams are fake calls from people claiming to be from a bank or other organization.
They aim to trick you into revealing personal information such as bank details, passwords,
etc...m~ ..!5

• Never give personal information to people who call you unexpectedly.


• Call the company back on a known number
• Always hang up if you're in any ~~ .

Don't be fooled - be smart!

13. (A) Has 15. (A) Phone scams are common.


(8) Did (8) Most calls are genuine.
(C) Was (C) Stay safe with our tips.
(D) Have (D) Contact us for more help.

14. (A) warning 16. (A) accident


(8) alerting (8) inconvenience
(C) notifying (C) trouble
(D) referring (D) doubt

13.0 @ @ @ 14. 0 @ @ @ 15. 0 @ @ @ 16. 0 @ @ @


Score ./16

ExTRA PRACTICE ONLINE


Go to www.pass-the-toeic-test.com for more grammar and vocabulary exercises to help
improve your score on the TOEIC test.

PART 6 TEXT COMPLETION 217


PART
7 READING COMPREHENSION

This part of the TOEIC" test consists of a series of short passages. The passages are
followed by a number of questions, each with four answer choices. You must choose the
best answer for each question.

Single passages
There are usually ten single reading passages. Each passage is followed by between two
and four questions. There are 29 questions in this section.

Double passages
There are two double reading passages. These are readings that include two related
passages. They are followed by five questions. There are 10 questions in this section.

Triple passages
There are three triple reading passages. These are readings that include three related
passages. They are followed by five questions. There are 15 questions in this section.

Questions test your general reading comprehension, as well as your understanding of


words and phrases in context. Sometimes for single passages you also need to choose
where to insert a missing sentence.

The passages cover a variety of different text types, such as advertisements, letters and
faxes, notices, e-mails and memos, forms and charts, and articles. Before each passage,
there is an introductory statement that specifies the type of passage you will read.

218 READING
TRY IT OUT
EXAMPLE 1 - a single passage

New leader takes control of GBK, Inc.


Today, GBK, Inc, confirmed the appointment to C.E.O. of Canadian business leader Spencer
McGregor. The construction giant elected McGregor due to his extensive experience and
previous record. McGregor previously saved Hansely Engineering from collapse, taking it out
of trouble to regain its former leading position in the industry. He replaces former C.E.O. Gary
Oldson, who was dismissed last month after reports of financial mismanagement. In recent
months, GBK has suffered a record drop in sales, with disappointing results for the last quarter,
However, the news of McGregor's appointment was welcomed by industry experts, causing the
stock price to increase 12 percent.

1. What is the main purpose of this article?


(A) To advertise a job opening
(B) To declare the retirement of a business leader
(C) To announce a new corporate executive
(0) To declare poor financial results
~---~ - - - -- -

The article begins by stating that GBK, Inc. confirmed the appointment to CEO, of Spencer
McGregor, and later refers to news of McGregor's appointment,

2. What happened to Gary Oldson?


(A) He moved to another department.
(B) He was recently fired.
(C) He retired from the company.
(0) He joined Hansely Engineering.
-------_._--
The article mentions he was dismissed last month. '
J

3. What can be inferred about GBK, Inc.?


(A) The company is not performing well.
(B) The company has few good managers.
(C) The company is owned by a Canadian firm.
(0) The company will soon expand.
- ---
_.- -- --
The article mentions a record drop in sales and disappointing results.
- _._- - ----- ~~._.~. - - .

PART 7 READING COMPREHENSION 219


EXAMPLE 2 - a double passage

Parham Printing, Inc.


Overseas Visitor Tour
Tuesday, August 21

9:00 A.M. - 10:00 A.M. Welcome & Introductory Lecture, Dr. Gange, C.E.O
Seminar Room A

10:00 A.M. - 11:30 A.M. Group A - Tour of Print Room, Mr. Walker, Print Manager
Group B - Tour of Warehouses & Packaging, Miss Rivera,
Operations Director

11:30 A.M. - 12:30 P.M. Distribution Challenges, Mrs. Atkins, Vice- President

12:30 P.M. - 1:30 P.M. Lunch in Main Cafeteria (with department heads)

1:30 P.M. - 3:00 P.M. Group A - Tour of Warehouses & Packaging, Miss Rivera,
Operations Director
Group B - Tour of Print Room, Mr. Walker, Print Manager

3:00 P.M. International Development Strategy, Dr. Nicholson, President


Seminar Room C

MEMO
To: Senior staff
From: Naomi Arakawa. Program Manager
Date: Aug. 17
Re: Tour next Tuesday

As you know, next Tuesday's Overseas Visitor Tour is a group from Far East Orient Printing, Inc.
There will be 20 senior executives representing over a dozen of the top printing firms in East Asia.
This is a huge opportunity for us to increase our services to the region.

Can I ask you all to please be prompt for all appointments, and keep to the schedule? You will
notice the length of the group tours is no longer two hours. This is because our visitors have to
leave at four o'c1ock. A bus will pick them up at Gate 0, so they will need ten minutes to get there.
Therefore, Dr. Nicholson, could you please try to complete your talk in 45 minutes? The last thing
to mention is that we may move lunch to the Green Room if there is space, as the main cafeteria
might be too noisy. I will let you know whether this is possible as soon as I hear.

Thank you for your cooperation. I am sure the tour will be a success.

220 READING
1. What is the main purpose of Naomi Arakawa's memo?
(A) To thank staff for their help with the tour
(B) To notify staff of changes to the tour schedule
(C) To encourage staff to go on the tour
(D) To ask staff to help organize the tour
- - -- -
Ms. Arakawa mentions changes in the length of the group tour and also the departure time, and
possibly lunch venue. (A) IS incorrect as the tour hasn't taken place yet. The tour is not for the staff
to participate in (C) and it is already organized (D).
. -

2. How much shorter is each group tour than originaliy pian ned?
(A) Fifteen minutes
(B) Thirty minutes
(C) One hour
(D) Two hours
- .. -
In her memo, Ms. Arakawa says the length 01 the group tours is no longer two hours. In the
schedule, the group tours are shown as lasting 90 mmutes (from 10:00 A.M. - 11:30 P.M.). They are
thirty minutes shorter.
. .

3. Where wili Mr. Walker take Group B in the afternoon?


(A) To the Cafeteria
(B) To the Warehouses
(C) To Seminar Room A
(D) To the Print Room
. .
In the schedule, Mr. Walker is shown to take Group B on a tour of the Print Room in the afternoon.

4. By what time should Dr. Nicholson aim to finish his talk?


(A) 3:00 P.M.
(B) 3:30 P.M.

(C) 3:45 P.M.


(D) 4:00 P.M.

In the memo, Ms. Arakawa ask Dr. Nicholson to fimsh his talk in 45 minutes. In the schedule, his
talk starts at 3:00 P.M. He should therefore finish by 3:45 P.M.

5. In the memo, the word "prompt" in paragraph 2, line 1, is closest in meaning to


(A) polite
(B) prepared
(C) patient
(D) punctual

Here, the word prompt means punctual or on time.

PART 7 READING COMPREHENSION 221


EXAMPLE 3 - a triple passage

,
NATIONAL LEAGUE FOOTBALL
I. Boston Cheetahs
vs. 'I,
Buffalo Hawks !
i

Supermax Stadium i
'I
Thursday October 9
Kick-off at 8.30 P.M. '!I,
I,i

Admission by ticket only 'i


. :.''':.''.''' '_,' ~.~-JJ
...:_~."-cc:--"~,,==~=~,.

From: Jai Chaudhry <Lchaudhry@memotex.com>


To: All staff <all_staff@memotex.com>
Subject: Tickets for the Thursday game
Date: Friday October 3

Dear all,

We have been given 20 free tickets for the playoff between the Cheetahs and the Hawks this
coming Thursday evening at the Supermax Stadium. If you would like to go to the game,
reply direct to me as soon as possible. Tickets will be given out on a first come, first served
basis. Please note, we can allow a maximum of two tickets per staff member.

Sincerely,
Jai Chaudhry
Human Resources

Brad 5:15 P.M.


Hey Ron. Do you want to go to the big game tomorrow evening? I know
you're a fan. I was given two free tickets at work!

Ron 5:18 P.M.


Woah! Are you kidding me? You bet! Those tickets are like gold dust.

Brad 5:21 P.M.


Yeah. I was lucky! Let's meet in the Roadrunner Sandwich Shop near South
Station. OK with you?

Ron 5:23 P.M.

Sure. What time?


Brad 5:25 P.M.
I'll go straight from work, so say around 6:30. We can grab a bite and get to
the stadium by 7:30. There should be a great atmosphere.

Ron 528 PM
Fine by me. See you then. Hey, thanks, man. I'll wear my Cheetahs jersey.

222 READING
1. How did Brad get tickets to the game?
(A) He took part in a prize drawing.
(B) He responded quickly to an e-mail.
(C) He went in person to Human Resources.
(0) He was rewarded for good performance .
.,_.~_ .. --- - --- - - -

In his e-mail, Jai Chaudhry asks staff to reply direct to me as soon as possible and
adds tickets will be given out on a first come, first served basis.
- ----

2. On which day were the text messages written?


(A) Wednesday
(B) Thursday
(C) Friday
(0) Saturday
-~~~----~--
The advertisement states the game is on Thursday. In his text, Brad asks
Ron if he would like to go to the game tomorrow evening.
~-'-- ---- - -
3. What can be inferred about Ron?
(A) He will travel to the stadium by train.
(B) He has known Brad for a long time.
(C) He works for Memotex.
(0) He supports the Boston Cheetahs.
- - -~.~
Brad texts Ron I know you're a fan and Ron says he Willwear his Cheetahs jersey .
. .. ~--~_
--~--
4. How early does Brad want to arrive at the stadium?
(A) A half hour
(B) An hour
(C) An hour and a half
(0) Two hours
- -- - -

I Brad wants to get to the stadium by 7:30. We can see from the
advertisement that the game starts at 8:30 P.M.

5. In the e-mail, the word "note" on paragraph 1, line 4, is closest in meaning to


(A) remark
(B) record
(C) notice
(0) show
~. ,,~ .._.'. - .. _- --
o
Here, the verb note means take notice.
• • _' __ ' __ '"

PART 7 READING COMPREHENSION 223


Directions: Read the texts. You will notice that each text is followed be several questions. For each
question, decide which of the four answer choices: (A), (B), (C), or (D), best answers the question. Then
mark your answer.

Questions 1-4 refer to the following letter.

Mr. Harno Katsuzawa


19 Delaware Court
Chicago, Illinois 60601

May 3

Dear Mr. Katsuzawa:

Thank you for your letter dated April 17. I am sorry that you are not satisfied with the outcome of your
application for registration in our Computing for Beginners course. As you know, this one-week course is
free, and therefore demand was very high. We felt we needed to admit a broad range of participants from
many different backgrounds.

Of the sixteen spaces available, half went to men and half to women. Of the men we admitted, three
are unemployed, two are retired, and the remaining three have part-time jobs. The course takes place
Monday through Friday from 10:00 A.M. until 1:00 P.M. Since you have a full-time job already, we were
unsure how you would manage to attend the course.

Thank you for your interest in the Chicago Lifelong Learning Center. I do hope we can welcome you into
a suitable course at some point in the future.

Sincerely,

Dr. Brenda stevens


Admissions Supervisor
Chicago Lifelong Learning Center

1. Why is Dr. Stevens writing to Mr. Katsuzawa? 3. Why is Mr. Katsuzawa unsuitable for the
(A) To confirm his attendance in a course course?
(B) To thank him for his application (A) He is retired.
(C) To offer him a teaching position (B) He works part.time.
(D) To explain the selection procedure (C) He works full-time.
(D) He is unemployed.
2. How long is the Computing for Beginners
course? 4. The word "admitted" in paragraph 2, line 1, is
(A) Five mornings a week closest in meaning to
(B) A total of 15 hours (A) agreed
(C) Two weeks (B) accepted
(D) Lunchtimes only (C) interviewed
(D) discussed

1.@ @ @ @ 2. @ @ @ @ 3. @ @ @ @ 4. @ @ @ @

224 READING
Questions 5-8 refer to the following e-mail.

0 E-mail

To: I undisclosed recipients


I
From: I CustomerRelations@PrcPub.com I
Subject: I Work in Action e-mail alert
I
Dear Customer:

I am pleased to inform you that the latest edition of "Work in Action" is now available. ---[1]---
You can download this issue at www.prc-publications.com. Those of you who have subscribed
to the print edition should receive your copy shortly. ---[2)---

The main feature this month is about financial support and funding. Diane Shaw, from Carpe
Financial Management, reveals how to secure the money you need to help your business grow.
Her informative article will help you to learn more about sources of funding, both regional
and national. ---[3]---

Also in this month's issue, you'll find a survey about "Work in Action." Your participation in this
survey is very important, as it will help identify ways to improve the magazine. Even if you have
not yet read "Work in Action," your views would be helpful. The survey should take no longer
than ten minutes to complete. ---[4]--- Responses received by September 15 will be entered
into a competition to win a notebook computer, a smartphone, or even a vacation to Florida!

Mark Perkins
Subscriptions Officer, PRC Publications

5. What is the purpose of this e-mail? 7. In which position marked [1]. [2]. [3]. and [4]
(A) To invite people to take a survey does the following sentence best belong?
(8) To announce a recent publication "All answers are of course confidential."
(C) To ask for financial support (A) [1]
(D) To recruit more employees (8) [2]
(C) [3]
6. Who is Diane Shaw?
(D) [4]
(A) A financial expert
(8) A business writer 8. Who is this e-mail intended for?
(C) A publisher (A) Finance managers of large companies
(D) A researcher (8) Customers who receive "Work in Action"
(C) Workers at PRC Publications
(D) Writers of articles for "Work in Action"

5.@ @ @ @ 6. @ @ @ @ 7. @ @ @ @ 8. @ @ @ @

PART 7 READING COMPREHENSION 225


Questions 9-11 refer to the following advertisement.

226 READING
9. According to the advertisement, who can 11. What must customers do if they want the web
benefit from the cheapest web saver rate? saver rate?
(A) Any customer who books online (A) Pay the total amount in advance
(B) Regular business clients only (B) Contact the Reservations department
(C) Anyone booking on a red date directly
(0) All new customers (C) Book within forty-eight hours of arrival
10. What is NOT indicated about the hotel? (O) Stay at least two nights
(A) The food is good.
(B) The location is convenient.
(C) Internet access is free.
(0) Corporate clients are welcome.

19'@ @ @ @ 10. @ @ @ @ 11. @ @ @ @ I

PART 7 READING COMPREHENSION 227


Queslions 12-16 refer to the following two e-mail messages.

To: MargaretJen nings@centralstationery.com.us


From: JulianHilper@aranlo.com.us
Dale: Ocl12
Subject: Prices and standards

Dear Ms. Jennings:


For the last five years I have purchased all of our office supplies from Central Stationery. Over that
time, I estimate the orders I have placed to value more than $25,000. I am writing to inform you,
however, that I will no longer be ordering supplies from your company.
I have found that the prices you charge are in some cases more than double those charged at
other stores. For example, last week I bought ten BestBrand X30 black printer cartridges for a total
cost of $290. I notice from their online shop that the same cartridges only cost $140 in OffMart. I
also bought three Denby filing cabinets, which are identical, but was charged different prices for
them; two are $55 while a third is $65. This makes no sense to me at all.

Following advice from my coworkers, I will therefore take our business elsewhere, and shop at
OffMart from now on.
Sincerely,
Julian Hilper
Office Manager

To: JulianHilper@aranlo.com.us
From: Desmond Prent@centralstationery.com.us
Date: Oct 19
Subject: Your complaint

Dear Mr. Hilper:


I am sorry that you feel dissatisfied with the prices we offer at Central Stationery. Ms. Jennings has
forwarded your complaint to me, and I would like to respond to the issues you raise.
1. Our policy is to stock items at the same prices, or as close as we can, as other office supply
companies in this area. The BestBrand X30 black printer cartridges you mention seeing on
OffMart's website are currently on special offer, and available only to customers who purchase
over $1000 of office furniture at the same time. You will notice that their regular price for these
cartridges is actually $32 each.
2. Regarding the Denby filing cabinets, two are an older model. They were the last two in stock
and were on clearance. The third cabinet you received is the new model, which appears the same,
but is of superior quality. This explains the difference in cost.
I do hope you continue to shop at Central Stationery. We value your business and look forward to
your next order with us.
Sincerely,
Desmond Prent
Customer Relations Manager

228 READING
12. Who is Julian Hilper? 15. How much does the new Denby filing cabinet
(A) A store manager at altMart model cost?
(8) A customer service agent for 8estBrand (A) $32
(C) A buyer for Central Stationery (8) $55
(D) An employee of Aranlo (C) $65
13. What does Mr. Hilper mostly complain about in (D) $140
his e-mail? 16. In Mr. Prent's reply, the word "value" in
(A) Low discounts paragraph 4, line 1, is closest in meaning to
(8) Poor quality (A) estimate
(C) Long delivery times (8) believe
(D) High prices (C) demand
14. How did Mr. Hilper find out the cost of altMart's (D) appreciate
8est8rand X30 black printer cartridges?
(A) A coworker told him.
(8) He checked altMart's website.
(C) He received a special alter coupon in the
mail.
(D) There was an advertisement in the local
newspaper.

12.@ @ @ @
13.@ @ @ @
14.@ @ @ @
15.@ @ @ @
16.@ @ @ @

PART 7 READING COMPREHENSION 229


Questions 17-21 refer to the following Web page, price list, and text message chain.

Let our chic and intimate function rooms bring a sense of beauty and joy to your wedding
reception. Experience the romance and magic of stars twinkling above you when dining in
our Conservatory, or enjoy the fabulous ocean views from our private outdoor terrace.

Your Tropical Bay wedding planner will make your wedding dreams a reality - overseeing
everything from invitation design to floral decorations. Our chefs will prepare the menu
of your choice, and customize the perfect wedding cake for you. Tropical Bay can also
recommend reputable photographers, dressmakers, and tuxedo rental services.

We have nearly 300 bedrooms, so your guests don't need to worry about driving home.

Reception halls Size Capacity (guests seated


at round tables)
Grand ballroom. ~ - 11-;. .. -,,1 .
- 4602 sq. ft. 420
Holyrood Room (with conservatory) 1824 sq. ft. 150
Dunedin Room (with private terrace) 1178 sq. ft. 80
Lochrin Room 784 sq. ft. 70

Tropical Bay accommodations


Price per room
per night

Deluxe 2-Bedroom Suite Two private balconies


Panoramic ocean view
$380
Luxury kitchen
Top floor
Standard room with King-size bed Private balcony
Ocean view $175
Refrigerator
Standard room with 2 Queen-size Private balcony
beds Ocean view $190
Refrigerator
Family room Sleeps 4
Crib available
Private balcony overlooking $220
tennis courts or swimming pool
Refrigerator

In addition, all rooms have: En-suite bathroom with shower, Air conditioning,
Satellite television, Telephone, free Wi-Fi (in public areas), In-room safe.

230 READING
Mia 9:12 P.M.
Hi Rosa. Just looking at the hotel where Joe and Alysha are holding their
wedding reception. Rooms are pretty expensive. Want to share?

Rosa 9.21 P.M.


Good idea. I'm a bit nervous actually. I won't know anyone there except for
you and Alysha. I don't like large crowds @

Mia 928 P.M.

Alysha told me the reception hall has a terrace, so we can spend our time
there. It'll be quieter.

Rosa 933 P.M.

Good idea. What about accommodations? It's not too late to book, is it?

Mia 9:39 P.M.


No, but we need to book now, if we want a cheaper room. One night is
enough, right? I'll book a room with two beds and pay with my credit card.
We can settle up at the hotel. How's that?

Rosa 9.43 P.M.


Great, thanks!

17. What is indicated about the Tropica! Bay Hotel? 20. How much will Rosa and Mia pay in total for
(A) It has recently been renovated. their hotel room?
(B) It is situated in a coastal area. (A) $175
(C) It accommodates up to 300 people. (B) $190
(D) It offers a tuxedo rental service. (C) $220
(D) $380
18. Which of the function rooms will Alysha and
Joe's wedding reception be held in? 21. What is indicated about Rosa?
(A) Grand Ballroom (A) She dislikes noisy environments.
(B) Holyrood Room (B) She would prefer they had booked earlier.
(C) Dunedin Room (C) She does not have many friends.
(D) Lochrin Room (D) She will pay her share when they meet.

19. Which facility is NOT a feature of every hotel


room?
(A) Private balcony
(B) Refrigerator
(C) Free Wi-Fi
(D) Lockbox
17.@ @ @ @
18.@ @ @ @
19.@ @ @ @
20.@ @ @ @
21.@ @ @ @

Score ./21

PART 7 READING COMPREHENSION 231


IMPROVE YOUR PERFORMANCE

In this section you will practice ways to improve your score on Part 7 of the TOEIC~ test.

These are the exercises you will cover:

Reading for Main Ideas ~ reading for overall comprehension; learning


how to skim texts; identifying the writer, the
topic, the audience, and the purpose

Reading for Details reading for specific information; learning how


to scan texts; identifying important points and
key information

Making Inferences ~ reading for information that is not directly


stated and drawing conclusions

Double Passages reading two related passages and referencing


information in both texts

Triple Passages ~ reading three texts of different genres and


relating information between them

In the TOEIC test, there are passages on many different topics, covering a variety of text types:
advertisements, forms, letters, faxes, e-mails, memos, tables and charts, schedules, instructions, notices,
news articles, etc. You will practice reading a wide range of text types.

As you work through Improve Your Performance, look back at your answers in the Try It Out section and
try to identify your strengths and weaknesses. Notice what problems you have, and focus on the areas
you need to improve.

232 READING
Reading for Main Ideas
In Part 7, most passages have at least one question that tests your understanding of the main idea.

TIPS I Reading for Main Ideas - Part 7 Reading Comprehension


• Look for topic questions: What is this article about? What does this letter mainly discuss?
• Look for purpose questions: What is the purpose of this e-mail? Why was this memo written?
• As you read, ask yourself: What is the passage about? What is the purpose of the passage?
Who is the passage written for?
• Remember, you do not need to read every word in order to understand the main ideas.
• You can often find information about the main idea near the beginning or end of a
passage.
• Don't worry about words you do not understand. Keep reading until you reach the end.
• Look for words or phrases that are repeated. They may be important.

EXAMPLE

To: All Faculty


From: The Dean's Office
Date: Monday, October 10
I would like to remina all of you that next Monday is our Founder's Day, readthe firstsentence
and therefore no classes will take place. However, lecturers and other
pickout somekeywords
teaching staff must still come in as we will be having a "Skills Improvement and phrases
Workshop" as part of our faculty development program. Your department
heads have organized sessions throughout the day, from nine througli five.

This is important, so please be here. I lOOKforward to seeing you all readthe lastsentence
on Monday.
Dr. Fisher, Dean

1. What does this memo mainly discuss?


The notice is from The Dean's Office to All Faculty.
(A) A national holiday It mentions a faculty development program for
(8) Class schedules lecturers and other teachers. which suggests
(C) Staff training training.
(D) Employee attendance

- . .- .
2. What is the main purpose of this memo?
The Dean says there are no classes next Monday
(A) To announce a new job opening
but staff must still come in for a workshop. The
(8) To encourage staff to attend a training Dean adds This is important, so please be here.
day
(C) To emphasize the need for better
qualifications
(D) To ask staff to work longer hours

3. What kind of institution is involved? The notice mentions lecturers and faculty. and is
(A) A library written by the Dean. All these terms are related to
(8) A training company universities.
(C) A sports facility
(D) A university

PART 7 READING COMPREHENSION 233


Reading for Main Ideas
Passage 1

Skim this e-mail, then immediately answer the questions. Check (,f) your answers. Do not look back at
the passage.

E-mail

From: I EryIWilliams1@FastNet.com I
To: I Lilly.Baker@maybridge.com I
Subject: I Leadership conference last Tue. I
Hi Lilly,

I would like to say "many thanks" to you and all the staff for making our conference at the
Maybridge Center such a great success. The event went extremely well, with lots of positive
comments coming in all the time. Many delegates mentioned what a fantastic venue the
Maybridge Center is, with excellent facilities and wonderful catering. All the staff were very
helpful and cheerful, too, which is so nice.

Would you please pass on our thanks to the whole team? I look forward to holding more events
in your great facility.

Kind regards,
Eryl Williams
Personal Assistant to Chief Executive Officer

1. What is this e-mail about? 3. What is the Maybridge Center?


[ 1 A recent conference [ J A school
[ 1 The results of a competition [ J A convention center
[ 1 A sporting event [ J A restaurant

2. What is the purpose of the e-mail?


[ 1 To book an event
[ 1 To ask for employment
[ 1 To thank an organization's staff

234 READING
Reading for Main Ideas
Passage 2

Skim this article, then immediately answer the questions. Check (.,I) your answers. Do not look back at
the passage.

Residents Say "Exciting" Development Not Welcome


Local residents in the town of Brookside gathered development. They included the landowners
last night to protest against plans for a new fast and representatives of The Pine County Business
food restaurant and gas station planned for the Forum, which is financing the development. Head
east side of town. "We feel it is unsuited to the of the fomm, Clark Debussy, claimed that studies
neighborhood," said Mayor Maddie Perrows. on the environmental impact have proven the
"There is no need or desire for it. It will mean more new development will have mainly positive effects
traffic problems. more noise, more pollution, and in the community. "We also expect that these
people are worried about the negative effect on facilities will attract more people to the area and
their property values." More than 200 residents boost local businesses," he insisted. However, the
came to the special council meeting, which didn't vast majority of residents remain unconvinced.
end until nearly midnight Friday. Of those who Officials from the fast food chain could not be
attended, only ten were in favor of the planned reached for comment.

1. What is the article mainly about? 3. Where was the article published?
[ 1 The need for more local facilities [ 1 In a college newsletter
[ 1 Opposition to a proposed development [ 1 In a local newspaper
[ 1 The result of some environmental studies [ 1 In a science journal
2. Why was the article written?
[ 1 To promote investment in local services
[ 1 To report on a community meeting
[ ] To persuade residents to back a plan

PART 7 READING COMPREHENSION 235


Reading for Main Ideas
Passage 3

Skim this fax. then immediately answer the questions. Check (.t) your answers. Do not look back at
the passage.

TransBank International

FAX To: Adam Wong


Number: 011-44-2072-9836428
Pages: 4
From: Michelle Bertram
Date: Jan 17

Urgent [.t] Please reply [ For review [] Please recycle [ 1

Dear Mr. Wong:

Thank you for your e-mail expressing interest in our International Savings Account. In accordance
with banking regulations, I must ask you to confirm some personal details before we can proceed.
Please find follO\ving an application form UY87, which I would like you to complete and sign. After
you fax this back to me, I will process your application.

I look forward to welcoming you as a customer.

Best regards,

MicheUe Bertram
Overseas Account Manager
,
1. What does the fax mainly discuss? 3. Who is Mr. Wong?
[ I Applying for a job [ ] A bank employee
[ I Banking regulations [ I A potential investor
[ 1 Application requirements [ 1 A personnel manager
2. What is the main purpose of this fax?
[ ] To recommend a service
[ I To request information
[ 1 To thank a customer

236 READING
Reading for Main Ideas
Passage 4

Skim this letter, then immediately answer the questions. Check (.f) your answers. Do not look back at
the passage.

Dr. Indira Gupta


IndiChemicals Ltd.
3487 Kasturba Road Cross
Bandra (East)
Mumbai
India - 402051

November 4

Dear Dr. Gupta,

Thank you for taking the time to visit Barlow Industries while you were on vacation here last month.
It was wonderful to meet you and learn of your research interests. I trust you have now returned home
safely.

You mentioned the possibility of working together on a research project. I have spoken with my
coworkers here,. and we agree this is an excellent idea. Your knowledge and experience in the field of fuel
cells and renewable energy is certainly of interest to us. I have therefore asked Julie Artens, our Special
Projects coordinator, to contact you regarding a research program that might interest you.

1. What is the letter mainly about? 3. Who is Dr. Gupta?


[ J Education opportunities [ 1 A scientific researcher
[ 1 Cooperating in research [ 1 A recruitment manager
[ 1 Travel arrangements [ 1 A politician
2. What is the purpose of the letter?
[ 1 To advertise a job vacancy
[ 1 To explain research aims
[ 1 To agree to work together

PART 7 READING COMPREHENSION 237


Reading for Main Ideas
Read this article and choose the most appropriate answer: (A), (8), (C), or (0), for each question.

PROFITS FALL FOR RETAIL GIANT


JACK GANDLEY, PORTLAND

July 15

Harney's, the state's top electronics retailer, has announced a fall in profits of 11.5% for
the first six months of the year. This represents an eye-watering $21 million loss. The retail
giant blamed poor consumer demand and a disappointing holiday season. Harney's C.E.O
Arthur Ledeal said, "Confidence among consumers has been low since the New Year, and
we don't expect it to pick up anytime soon. However, the structure of the industry is strong
and we hope to recover toward the end of the year." He added that Harney's intends to close
five of its stores across Oregon in the coming months. This is part of a recovery plan that is
expected to reduce overheads by $10 million over the next three years. The company's stock
price fell 14% after the announcement.

1. What is this article mostly about? 4. What does Harney's plan to do to recover?
(A) Portland's economy (A) Leave Oregon
(8) A large retail company (8) Cut costs
(C) Low stock prices (C) Increase demand
(0) Consumer confidence (0) Issue more stock

2. What is the purpose of this article? 5. What is suggested about Harney's?


(A) To announce the closure of several (A) The company is going through a difficult
stores time.
(8) To show the latest retail statistics (8) The company is enjoying strong growth.
(C) To analyze a company's post-holiday (C) The company is about to collapse.
sales (0) The company has a strong customer
(0) To report a company's performance service record.

3. Where was this article published?


(A) In a store leaflet
(8) In a company brochure
(C) In a business magazine
(0) In a college newsletter

1. (6) @ @ @ 3. (6) @ @ @ 5. (6) @ @ @


2. (6) @ @ @ 4. (6) @ @ @ Score ./5

238 READING
Reading for Details
In Part 7, you need to show that you can identify and understand important points in the texts you read.
Questions about details are the most common type of question in this part of the test.

TIPS I" Reading for Details - Part 7 Reading Comprehension


.;. Look for Wh- questions (e.g., questions beginning with What, When, Why, How, etc.)
-. "Before you read, make sure you know what information you need to read for.
',,"
•. Look for any headings or titles to help you find the part of the passage you need.
',. Don't read every word. Focus only on finding the information you need.
• Don't worry about words you do not understand.
• When you find the part you need, read more carefully. Identify any key words or phrases .
• . As soon as you have found the information you need, stop reading.
,',

EXAMPLE

ARGO Summer Conference Agenda


Thursday, July 10
Lookfor any keywords
12:15 Lunchtime networking relatedto the information
1:00 Lecture: The Future of Technology, Dan Swanson (IT Director) you needto find.
2:00 Panel discussion: Opportunities in Europe
3:30 Lecture: Strategic Management, Hye Kim (Operations Manager)
4:30 Coffee break Whenyou find the right
4:45 Lecture: Innovation, Hans Peterson (Marketing Director) part of the text,read
carefully.
5:45 Panel discussion: Growth in Asia
6:30 Closing speech, Nigella Coopers (C.E.O.)
7:00 Farewell Dinner, Function Room 1

1. When is the Marketing Director's afternoon


. After you find Marketing Director in the program,
lecture?
I you see Lecture and the time 4:45.
(A) Immediately after lunch .. .-
(B) Around 4:00 P.M.
(C) A quarter to five
(D) After the coffee break

2. What talk is Dan Swanson giving? After you find Dan Swanson In the program, you
(A) The Future of Technology see the title of the lecture The Future of Technology.
(B) Growth in Asia
(C) Opportunities in Europe
(D) Innovation

3. Who is Nigella Coopers? -- --- - -- -- - -~ - - -

After you find Nigella Coopers in the program. you


(A) Operations Manager
see her Job title given as C.E.O. ,
(B) Marketing Director - - - - -- -_.

(C) I.T. Director


(D) Chief Executive Officer

PART 7 READING COMPREHENSION 239


Reading for Details
Passage 1

Read the first question. Then scan this letter to find the answer. Only look for the answer to the question.
When you find the information you need, write the answer in the space provided. Then go on to the next
question. Answer all the questions as quickly as you can.

1. What is Ms. Gillingham's job?

2. When do the Adult Learning courses take place?

3. What course is Mr. Courtney most interested in?

4. What two documents does Mr. Courtney request?

5. How did Mr. Courtney hear about the discounted fees?

6. Who qualifies for discounted fees?

Admissions Officer 47 Ipswich Road


Redhill College Colchester
Westacre Drive Essex
Colchester C028GR
Essex COl 7SR

May 14

Dear Ms. Gillingham,

I recently read an advertisement for Adult Learning courses at Redhill College and am writing to request
further details. [ understand these courses are on weekday evenings, which is fine for me, as I work
during the day. I am especially interested in your Skills for Work program. Do you offer any courses in
Accounting? [f not, then [ might be interested in a Computer Skills course. I would appreciate it if you
could please send me a brochure. A friend told me that the fees are reduced for people with low incomes.
Can you please include a price list?

Yours sincerely,

Mike Cou.rcney

240 READING
Reading for Details
Passage 2

Read the first question. Then scan this notice to find the answer. Only look for the answer to the
question. When you find the information you need, write the answer in the space provided. Then go on
to the next question. Answer all the questions as quickly as you can.

1. Where does "The Learning Brain" lecture take place?

2. What is the title of Professor Kelly Walters' lecture?

3. Which lecture lasts the longest?

4. Who should you ask for a ticket?

5. What time does Dr. Silvia Petronelli's lecture finish?

6. Who can go to these lectures?

The Connaught Lecture Series Astrobiology - the search for life in the
Chollerlord University of Science universe
Chollenord University is proud to announce this Lecture Room 12, Department of Physics
year's series of Connaught lectures. Once again.we Astrobiology is a new and exciting field of science.In
are delighted to bring three top scientists to the this stimulating lecture. discover how the first stars
university.Pleasecontact our Academic Program and galaxies were formed. Join Dr. Silvia Petronelli on
Manager.Sally Erskine,as soon as possible to reserve a journey to other planets and moons in the hunt for
a seat Tickets are issued free of charge on a strictly alien life.We are not alone'
"first come, first served" basis. March 17,11:30 A.H.- 1:15 P.M.

The Learning Brain


Living Energy
Melrose Hall
Patten Building
Dr. Thomas Gingrich will explain the latest research
Learn how we can use plants to create renewable
into how our brains work. Every second, we process
fuel.Watch real-life experiments, and be amazed as
huge amounts of information. But how do we do this.
Professor KellyWalters demonstrates new sources of
and what happens when things don't function as well
energy for the future.
as they should? Come find out!
March 23, 10:00 A.M.- 11:30 P.M.
March 3, I :30 P.M. - 3:30 P.M.
Only members of the university may attend. We
regret that we cannot admit family members or
friends to any lecture.

PART 7 READING COMPREHENSION 241


Reading for Details
Passage 3

Read the first question. Then scan this e-mail to find the answer. Only look for the answer to the question.
When you find the information you need, write the answer in the space provided. Then go on to the next
question. Answer all the questions as quickly as you can.

1. When did Claire return from her recent business trip?

2. Why has work on the Franklin Project stopped?

3. Why did Claire not reply to Rupert sooner?

4. Where is Claire going tomorrow?

5. When will she return?

6. What time does she say Rupert can call her?

E-mail

From: I Claire Ling


To: I Rupert Lee
Subject: I The Franklin Project

Rupert:

I am sorry it has taken me so long to reply. I have been on a business trip to Seoul and just
got back yesterday. Anyway, to answer your query, the Franklin Project has been going well,
but we are still waiting for payment for Phase Three. I have told them we cannot proceed to
the next phase until we receive payment in full for the work we have done. I'll be away again
tomorrow until Friday. I have to go to Beijing to give a talk. If you want to discuss this, please
call me at four o'clock this afternoon.

Claire

242 READING
Reading for Details
Passage 4

Read the first question. Then scan this advertisement to find the answer. Only look for the answer to the
question. When you find the information you need, write the answer in the space provided. Then go on
to the next question. Answer all the questions as quickly as you can.

1. What does Cam Call Computing Solutions specialize in?

2. Which customers can expect emergency help in less than 60 minutes?

3. Which service costs $250 per month?

4. What should you do for more information?

5. How much does the "Back to Basics" software training cost per hour?

6. Which service do you NOT have to pay for?

CAM CALL COMPUTING SOLUTIONS


2036 Monroe Boulevard
Orlando, FL 34545

At CamCall Computing Solutions we Technical assistance


understand technology. That's why we can help For as little as $2000, we can design a computer
you with a wide range of services. system to suit your exact requirements. What's
Web design more, we'll maintain your computer system so
We know what makes a website successful for it operates at maximum capacity. Imagine, no
an organization. Book a free consultation with more computer problems! We are specialists at
our web design team and get a professionally removing viruses and spyware. Monthly "PC
designed website for your company from just Total Care" fee - $250.
$1500. Emergency help
Software training Our engineers will solve any technical issue
Save money and time! We will train your staff you may experience. For customers in Orlando,
to use the latest software packages quickly and we guarantee to respond within one hour.
efficiently. Our "Back to Basics"courses cost Minimum service call charge of $175 applies.
$500 for ten hours. Parts and labor not included.

Call liS to discuss your requirements: 800-555-8957 (Hours: Mon. - Fri. 9:00 A.M. - 5:00 P.M.)

PART 7 READING COMPREHENSION 243


Reading for Details
Read this form and choose the most appropriate answer: (A), (B), (C), or (0), for each question.

BENDIX FURNITURE ESTIMATE


82 Cherry Lane, Biggleswade • Bedfordshire SG21 8PY • Tel: 01767 • 438798 I Fax:0 1767 • 438799
Dear Mr.Ames.
Please nnd below details of the office products you expressed an interest in during your visit to our
showroom last Friday.As discussed,we have applied the maximum discount on each item wherever possible.

Shipping: Free (1"0 charge for delivery on orders over l.1 000.)
Expected delivery times: Approximately 5 days after order is received.
Returns policy A cancellation fee of 20% of the order (ost will be applied in the event any items are retumed to ~s after delivery.
All goods are guaranteed for a period of one year from delivery.

We look forward to receiving your order.


Charlie Bryson
SalesTeam

1. Which item has the largest discount? 4. Why is there no charge for shipping?
(A) Viper metal storage cabinet (A) There is a special promotion.
(B) Oak bookcase (B) The order exceeds a certain amount.
(C) Milano executive chair (C) All items have free shipping.
(0) OfficePro conference table (0) Mr. Ames negotiated this separately.

2. How soon after ordering will the items be 5. How much is the order without VAT?
delivered? (A) £662.74
(A) The next working day (B) £926.25
(B) Within three days (C) £3313.70
(C) Within a week (0) £3976.44
(0) Over one week

3. When did Mr. Ames visit Bendix furniture?


(A) The previous day
(B) Over the weekend
1.@ @ @ @ 4. @ @ @ @
(C) At the end of last week
2.@ @ @ @ 5. @ @ @ @
(0) Sometime last month
3.@ @ @ @ Score /5

244 READING
Making Inferences
In Part 7, you occasionally need to show that you can understand things that are not directly stated in
the passages you read. Questions that test your ability to make inferences in a reading passage require
you to draw conclusions about information you read. You may also have to make connections between
information in different parts of the passage, or between two passages.

TIPS r-- ~~ki.ng Inferences-Part :.;R'eading'Compreh'ension


."', Look
~~ - for questions such as Which of the following can be inferred from this e-mail? Which
-

~•. ofthese statements is probably true? What is implied about...?


~.~ As you read, think about the meaning "behind" the words. Pay special attention to words
{" and phrases with positive and negative meanings.
','
., .Try to make deductions as you read, and draw conclusions; for example, about the
..f_. relationship between the two writers, or what might happen next.
: , . Consider the whole passage before making any inferences.
~~ I", _' • __ -. " __ 1_._ •

EXAMPLE

Date: March 8
To: sand raben nett@onenet.org
From: sales@uniprint.com
Subject: Order no. 98434 a long time
Dear Ms. Bennett:
It is now four weeks since we dispatched your order of 300 color brochures
(Ref. no. 98434). We hope you are happy with the quality and the service we invoicestili unpaid
have provided. Assuming this is the case, may I please remind you to ay the
hasaskedbefore
invoice dated Feb. 7 at the earliest opportunity? This is the third e-mail I have
sent regarding this matter. If we do not receive payment from you before Friday,
March 12, we will be forced to take further action. losing patience

Regards,
Beatrice Franklin (Accounts Receivable)

1. Which of the following can be inferred from


---_.. - ----- -
The e-mail seeks to remind the customer
this e-mail? to pay the invoice, which was sent on Feb.
(A) The brochures are poorly printed. 7. The date now is March 8.
(B) Ms. Bennett has been on vacation. -~. '-

(C) Payment for the brochures is overdue.


(D) Uniprint is a local company.

2. How does Ms. Franklin feel about the


Ms. Franklin says this is the third e-mail
situation?
to ask for payment, and suggests further
(A) Frustrated action may be necessary if payment is not
(B) Relieved made soon.
(C) Optimistic ~~.- -~

(D) Regretful

PART 7 READING COMPREHENSION 245


Making Inferences
Passage 1

Read this notice. Then answer the questions. Check (,f) your answers.

REMINDER
Will all staff who use the conference room for internal meetings please remember to take their
tea and coffee cups away with them afterwards. I have noticed in recent weeks that more and
more dirty cups are being left on the table and windowsill. This can't go on. Please remember this
makes a poor impression on our clients, who we also meet in this room.

Thank you for your cooperation.


Susan Shaw

1. Who most likely wrote this notice?


[ J An important client
[ 1 An office cleaner
[ 1 The office manager
2. Which of the following can be inferred from this notice?
[ ] The issue is getting worse.
[ 1 Customers have been complaining.
[ 1 There are too many internal meetings.
3. What does Ms. Shaw imply?
[ 1 Staff should meet in another room.
[ 1 The current situation is bad for business.
[ 1 No drinks should be taken into the conference room.
4. What does Ms. Shaw mean when she writes, "This can't go on"?
[ 1 Meetings are a waste of time.
[ 1 The situation has to change.
[ 1 There is no way to make progress.

246 READING
Making Inferences
Passage 2

Read this letter. Then answer the questions. Check (.1') your answers.

HANBARGER ALPINE RESORT


13 Mulberry Park Avenue
Mount Redmont, British Columbia V7L 2FP

September 23

Mr. Petain
234 Oak Valley Drive
SE Calgary, T4K 9B5

Dear Mr. retain:

Thank you for your letter expressing an interest in working as a ski instructor at Hanbarger Alpine
Resort. Although the position has only been advertised a week, the response has been overwhelming.
We have so far received many applications, including from instructors who have many years' experience
and are extremely well qualified. As you only have a Levell qualification, we regret that we are unable to
offer you a position at this time.

Sincerely,

Mary Sharlow
Human Resources

1. What can be inferred about Mr. Petain?


[ J He recently applied for a job.
[ 1 He is a graduate student.
[ 1 He works near Hanbarger Alpine Resort.

2. Which of the following can be inferred from this letter?


[ ] Many ski instructors are unqualified.
[ 1 Mr. Petain has trained as a ski instructor.
[ 1 Hanbarger Alpine Resort has no job vacancies.

3. What does Mary Sharlow imply about Mr. Petain?


[ 1 He should submit another application next year.
[ 1 He needs to achieve better qualifications.
[ 1 His qualifications are not appropriate.
4. What does Ms. Sharlow mean when she writes, "The response has been overwhelming"?
[ 1 She gets emotional thinking about it.
[ 1 Many more people applied than they expected.
[ I There are too many applicants for the jobs available.

PART 7 READING COMPREHENSION 247


Making Inferences
Passage 3

Read this online chat discussion. Check (,f) your answers.

Lee Harveys 940A.M.


Hey everyone. Thanks for helpin9 out. So, Assembly Line 4 is down and we need to
get the GTexorder out. Any suggestions?
Richard Ng 941 A.M.

I can get a team together and head down there to take a look. Maybe in half an hour?
Kim Sook 941 A.M

How long will it take to repair?


Richard Ng 9:41 to,M

No idea until the guys see which machine it is.


Lee Harveys 942 A.M.

I guess we can wait and see.


Kevin Danson 942 A.M.

No way! I promised them they would have the order by the end of the day.
This is GTexwe're talking about. We can't risk it. We need to swap to another Assembly line.
Paul Athers 943 A.M.

We could use nO.6. It was offline yesterday for cleaning, but Control Room says
that's finished so ...
Kim Sook 943 A.M.

Great! I'd say we go with that.


Kevin Danson 943 A.M.

Sounds good to me.


Lee Harveys 944 AM.

OK. Paul, can you get that sorted asap? Richard - please send your team to 4 anyway.
We need it back online.

1. Where most likely do the speakers work?


[ ] In a laboratory
[ J In a factory
[ lin a hospital

2. At 9:42 A.M., what does Mr. Danson mean when he writes, "This is GTex we're talking about"?
[ 1 GTex is a very important client.
[ 1 Do not confuse this client with another.
[ 1 They need to focus on the customer.
3. In which department does Richard Ng most likely work?
[ 1 Sales
[ 1 Finance
[ 1 Maintenance
4. Which of the following can be inferred from the discussion?
1 The situation is very urgent.
1 This difficulty happens often.
1 The repair may take a long time.

248 READING
Making Inferences
Passage 4

Read this text message chain. Check (.1') your answers.

Bill Mathers 2.12 P.M.


Hi Vernon. I just got here and found I forgot all the
brochures. and my notes I Are you in the office?
Vernon Rees 2:12 PM.
Oh no! Yes. I'm here. Are they on your desk?
Bill Mathers 2.13 P.M.
Try on the right, near the phone. Everything's in a
black folder, quite thick.
Vernon Rees 2.13 P.M.
Got iti Do you want me to mail it?

Bill Mathers 214 P.M

Yes, to the hotel. The address is in the inside front


cover. Use TransEx.
Vernon Rees 2.14 P.M.
Will do.

Bill Mathers 2.15p.M.

Thanks. Got to go. Flight's boarding.


Vernon Rees 215 P.M.

OK. Have a good one.

1. Where most likely is Bill Mathers?


[ lin an airport
[ l With a client
[ 1 In a hospital
2. What kind of organization is TransEx?
[ ] A courier service
[ 1 A taxi company
[ 1 A shipping line

3. What can be inferred about Mr. Rees?


1 He is a personal secretary.
1 He works with Mr. Mathers.
1 He is leaving the office soon.
4. At 2:15 P.M., what does Mr. Rees mean when he writes, "Have a good one"?
1 Stay positive.
] Have a nice stay.
1 Enjoy your trip.

PART7 READING COMPREHENSION 249


Making Inferences
Read this information and choose the most appropriate answer: (A). (B), {el. or (O),
for each question.

Great Business Leaders of the Past


FRANK BICKERSTAFF
Sun ValleyPress, $15.95

Frank Bickerstaff is well known for his profiles of business leaders. This is his latest
publication on the subject. and here he looks at prominent businesspeople from -
you guessed it - the US once again. Famous names such as Henry Ford. Bill Gates.
Jack Welch (the youngest-ever C.E.O. of General Electric). Andrew Carnegie. John
D. Rockefeller, and W.K. Kellogg are covered. Bickerstaff certainly goes into detail
about the characteristics needed to become successful in business. Qualities such
as the ability to learn from mistakes. motivate people, and take risks are thoroughly
investigated. However, I cant help but feel he missed a chance to look beyond the
borders of the US and search for inspiration elsewhere. Overall. it is a good read for
those who are new to the subject, but for those who are already familiar with the top
ten he chose it is a little lacking.
Brad Ellington

250 READING
1. Which of the following can be inferred about 3. Who most likely is Brad Ellington?
Frank Bickerstaff? (A) A book reviewer
(A) He is an excellent writer. (B) An employee of Sun Valley Press
(B) His books are best-sellers. (C) A successful businessman
(C) He has written on this topic before. (D) An expert in international business
(D) He is a successful businessman.
4. How does Brad Ellington probably feel about
2. What does Brad Ellington imply about the the book?
book? (A) It is exciting to read.
(A) Anyone who wants to own a business (B) it is a Iittie disappointing.
should read it. (C) It is badly written.
(B) It covers the most important business (D) It is reasonably priced.
leaders.
5. What does Brad Ellington mean when he
(C) Everyone interested in business will
writes, "It is a little lacking"?
enjoy it.
(A) Certain facts are incorrect.
(D) It focuses too much on American
(B) The book is not finished.
business leaders.
(C) It is not as good as it could be.
(D) Some sections shouid be longer.

1.0 @ @ @ 4. 0 @ @ @
2.0 @ @ @ 5. 0 @ @ @
3.0 @ @ @ Score ......../5

PART 7 READING COMPREHENSION 251


Double Passages
EXAMPLE

Temporary Cashier THE OWL'S NEST BISTRO


Salary: $20 per hour
Location: Augusta, Maine
Job term: Temporary, Full-time
Reference: 3459860
Contact: Vernon Marshall, Owner

Are you looking for a busy and varied role? Do you enjoy working in a fun but challenging
environment? The Owl's Nest Bistro is a successful family-owned business. For 25 years, we
have been Maine's biggest seafood and fish restaurant, serving up to 500 customers a day. We
are looking for an experienced cashier. You must be flexible and positive, and you need to have
excellent communication skills. Plus, since we also cater for corporate clients, you must have an
appetite for providing top-quality customer service!

Full training will be given. However, please note that this position is temporary. We expect the
position to last six months.

We need someone who is reliable, intelligent, and loves hard work. If you are interested in
applying, e-mail your resume and a cover letter, including your current salary, to Vernon Marshall.
Could this be the perfect opportunity for you?

From: briannajackson@maineonline.com
To: Vernon. Marshall@owlsnestbistro.com
Subject: Temporary cashier [Reference: 3459860].

Dear Mr.Marshall:
I am writing in response to your advertisement in Maine Business Monthly for a temporary cashier.
I have five years' experience at Lee Brothers, Inc. here in Augusta, where Iwork part-time as a
cashier. This has given me strong communication skills. I am used to managing several tasks
at once, and work well under pressure. I also have excellent customer service capabilities. I am
flexible, and have a good sense of humor, too.
I believe my skills make me a good match for the Owl's Nest Bistro. Iwould welcome the
opportunity to work on a temporary basis at your restaurant. Ifyou feel I might be suitable, I
look forward to coming in for an interview and telling you more about my abilities and how I can
contribute towards your establishment's success.
Sincerely,
Brianna Jackson

252 READING
1. Who would be most interested in the
. The position is temporary and will/ast six months .
advertisement? . Experience is necessary, so (A) is incorrect. (C)
(A) Someone wanting to start a new career. . confuses the job of cashier with banking. (0) is
(B) People looking for short-term incorrect as the job is full-time.
employment.
(C) Bank staff living in Augusta, Maine.
(0) Anyone looking for part-time work.

2. What is suggested about Ms. Jackson?


(A) She has the necessary experience.
(B) She is unhappy in her current job.
(C) She works full-time at the moment.
(0) She is overqualified for the position.

3. What is indicated about the Owl's Nest Bistro?


(A) It is part of a large group.
(B) It was recently renovated.
(C) It has a successful history.
(0) It serves all types of food.

4. Which quality that Ms. Jackson mentions is


(A), (C). and (0) are all mentioned'in the ~d. In her
NOT requested in the advertisement?
e.mail Ms. Jackson wntes she has a good sense
(A) Excellent customer service skills
(B) A good sense of humor I of humor, too.
- ._------ ---
(C) Strong communication skills
(0) A flexible approach

5. In the e-mail, the word "contribute" in --------_._. - -- ,


The verb contribute has all these meanings,
paragraph 3. line 4, is closest in meaning to
but we can tell from the context it is used here
(A) donate
meaning help (to cause or bring about).
(B) subsidize
(C) help
(0) supply

PART 7 READING COMPREHENSION 253


Passage 1/2

To: Gyles.Jones@studi0435.co.us
From: emma12@GeoTraining.co.us
Date: Wednesday, July 17
Subject: Course confirmation

Dear Mr. Jones

Following our recent phone call, please find attached details confirming your participation in our
"Dynamic Sales and Marketing" summer school program.
I would be grateful if you could acknowledge receipt of this e-mail. Please note that since you are
registering very late, you need to pay an extra $50 "priority registration" fee. Please add this to the
course fee and pay immediately. Our bank details are in the attached document. Because I am
away from the office until Monday, please e-mail our course planning team at courseplanning@
GeoTraining.co.us if you need further assistance. Alternatively, please speak to the Academic
Advisor for your course, Brian Sharpe, on Monday morning.
I hope you enjoy the course.

Best wishes
Emma Burton, Course Manager
GeoTraining, Inc.

GeoTraining, Inc.
Dynamic Sales and Marketing
Summer School

Location: The Hoover Building


Date: July 22 - July 26
Entry requirements: No formal entry requirements
Subject area(s): Sales f Marketing
Course description: This comprehensive course covers account management, telephone sales.
advanced selling techniques, and sales negotiation skills.In addition, you will
learn how to develop an effective and powerful marketing strategy. The
latest trends in marketing (such as viral marketing) will also be covered.

Assessment: Course participants will complete a short assessment.Successful candidates


will receive a GeoTraining Certificate ofTraining award.

Resident fee: $3.950


Application status: Approved

Bank details:
Bank: Hanubo International Account: Geo Training. Inc. Sort Code: 30-345-34 Account No: 098450864365

254 READING
Main Ideas
Skim the e-mail and notice. Then immediately answer the questions. Do not look back at the passages.

1. Who most likely is Mr. Jones? 2. Who is the notice intended for?
(A) A bank employee (A) Anyone interested in sales and marketing
(B) A course applicant (B) Entrants to a business competition
(C) A training manager (C) Recent employees at GeoTraining, Inc.
(D) An event planner (D) Candidates who have applied for a job

Details/Inferences
Read the first question. Then scan the e-mail and notice to find the answer. Only look for the answer to
the question. When you find the information you need. write the answer in the space provided. Then go
on to the next question. Answer ali the questions as quickly as you can.

1. Who is Brian Sharpe? .

2. How many days does the course last?

3. What do participants who pass the course receive?

4. If necessary, who can Mr. Jones contact before Monday?

5. What is the total amount Mr. Jones needs to pay?

6. What is Emma Burton's job? .

7. Where wili the training take place? .

8. When is payment due? .

9. To which bank should Mr. Jones send payment?

10. Why does Mr. Jones need to pay a "priority registration" fee? .

11. What qualifications are needed to enroli in the course?

12. How soon will the course begin? .

PART 7 READING COMPREHENSION 255


Passage 3/4

Gianna Rebecci
Human Resources Manager
Glyson, Inc.
154 Fifth Avenue
New York,NY, 10018 Tuesday, September 30

Dear Mrs. Rebecci:


With regret, I am writing to inform you that I feel unable to continue in my role here. Since the
new management changes came into effect last spring, I find that I am working much longer hours
than I am comfortable with. This has obviously affected my personal and family life. In addition,
staff cuts in the Research Division mean I now have far more responsibility and yet my salary has
remained the same. My second request for a raise has just been denied.
I have enjoyed my five years with Glyson. However, I hope you can understand my reasons for
leaving this position. I believe that this decision is in the best interests of my family and my career.
I will, of course, work the necessary one-month resignation period, making October 28 my last
working day here.
Sincerely,
Ian Ronson
Ian Ronson
Senior Scientist

MEMO
To: Ian Ronson
From: Gianna Rebecci
Re: Your letter

Dear Ian:

I was disappointed to read your letter. which I received yesterday. As you know, when I arrived at
Glyson in the beginning of April, the company was in terrible shape. I have tried to bring us up-to-date
so we are more competitive. This has meant staff reductions (including three members of your division).
As a result, we all have to work harder.

I am confident the measures I have taken will bring success. I am sorry you feel unable to be part of this
success as we move forward, but I respect your decision. I will make arrangements for your coworkers
to take over your responsibilities after your departure.

Regards.
Gianna

256 READING
Main Ideas
Skim the e-mail and memo. Then immediately answer the questions. Do not look back at the passages.

1. What is the main purpose of Mr. Ronson's 2. Why is Ms. Rebecci writing to Mr. Ronson?
letter? (A) She is trying to change his mind.
(A) To complain about staff cuts (B) She is questioning a decision.
(B) To ask for a pay raise (C) She is accepting his resignation.
(C) To arrange a meeting (D) She is expressing support for his opinion.
(D) To resign from his job

Details/Inferences
Read the first question. Then scan the e-mail and memo to find the answer. Only look for the answer to
the question. When you find the information you need, write the answer in the space provided. Then go
on to the next question. Answer all the questions as quickly as you can.

1. What company does Mr. Ronson work for?

2. When did Ms. Rebecci receive the letter? .

3. How many times has Mr. Ronson asked for a pay raise?

4. How long has Ms. Rebecci worked at Glyson, Inc.? .

5. Why is Mr. Ronson unhappy to be working longer hours?

6. What department does Mr. Ronson most likely work in?

7. How long has Mr. Ronson worked at Glyson, Inc? .

8. How much notice is Mr. Ronson required to give? .

9. How many members of the Research Division have been made redundant? .

10. On what date will Mr. Ronson leave the company? .

11. Why does Ms. Rebecci say redundancies were needed? .

12. Who will take over Mr. Ronson's job when he leaves? ...

PART 7 READING COMPREHENSION 257


MINI T~SJj Double Passages

o E-mail

From: I Enquiries@USTl.org
To: I Sanjaya.Murthy@edgeexports.com
Re: I ETF grant

Dear Sanjaya:
As we discussed when you called, the government has cut our budget for the coming year.
This has resulted in fewer grants, and in some cases the level of grant has also been reduced.
As you will see from the attachment, your industry has been the hardest hit.
Nevertheless, as this is your first time attending ETF, I think it is worth submitting an application.
You MUST read the terms and conditions before applying for a grant to confirm you qualify for
support. I would also encourage you to oniy apply for a grant if you are sure you will attend. In
the past, companies have applied for grants and then changed their minds at the last minute.
This obviously means that others miss out on the chance to go.
Anyway, if you decide to go ahead, I look forward to receiving your application for funding.
With kind regards,
Gladys
Ministry of Trade & Investment

Eastern Trade Fair

Overview of Government Cuts

Maximum funding per company (US$) by industry sector

2500
D Last year
• This year
2000

1500

1000 -

500

o
Leisure Financial Computing Clothing Medical Construction

258 READING
1. What does Gladys encourage Sanjaya to do? 4. Why does Gladys believe Edge Exports might
(A) Miss this year and apply for next year succeed in getting funding to attend ETF?
instead (A) It has good connections with the
(8) Wait until other companies submit their government.
applications (8) It is a major exporter in the region.
(C) Apply even if Edge Exports may not go to (C) It has not been to the trade fair
ETF before.
(D) Read another document before applying (D) It is a well.established company.
2. How much support can clothing companies 5. What industry sector does Sanjaya Murthy
receive to attend the next ETF? work in?
(A) $500 (A) Leisure
(8) $1,000 (8) Financial
(C) $1,500 (C) Clothing
(D) $2,000 (D) Construction

3. The word "confirm" in paragraph 2, line 2, is


closest in meaning to
(A) check
(8) inquire
(C) prove
(D) accept

1.@@@@ 4. @ @ @ @
2.@ @ @ @ 5. @ @ @ @
3.@ @ @ @ Score ./5

PART 7 READING COMPREHENSION 259


Triple Passages
EXAMPLE

Cream patent leather

open-toe

platform sole

Made in Italy

Sizes 5-8

$80

To: All staff <all_staff@westendappliances.com>


From: Eleanor Jarvis <ejarvis@westendappliances.com>
Date: April 13
Subject: Shoes for sale

Hi all,
Before I advertise these shoes (see attached) on an auction site, I thought I'd ask if any of you would like
to buy them from me. I bought them to wear at my brother's wedding last week, and have worn them only
on this one occasion. I really like them, but they're too high for me and I was wobbling all day! They're
size 7'h and are a very narrow fitting. I'm happy to sell them at half the price I paid for them. If you're
interested, please let me know. I have them in the office with me, so you're welcome to try them on.

Sincerely,
Eleanor Jarvis
Accounts Department

Jana April 13. 10:23 A M.


Hi Eleanor. I got your e-mail. Yes, I'm interested! They're my size exactly, and I like the look. I'm working at home
this morning, but I'll drop by this afternoon and try them on. Is that OK?

260 READING
1. Which feature of the shoes is NOT mentioned
--- .-------
The ad states that the shoes are Made in Italy, but
in the advertisement? does not name the manufacturer. They cost $80
(A) The manufacturer ! (6), are leather (C), and the color is Cream (D).
(B) The price -----_. -

(C) The material


(0) The color

2. What does Eleanor suggest about the shoes? In her e-mail, Eleanor writes she was wobbling l
(A) They were too expensive. (meaning walking unsteadily) all day, so we can
(B) They are not the right size. infer the shoes were uncomfortable to walk in. She I
(C) They are not very popular. writes the shoes are too high for me, but this does
(0) They are difficult to wear. not mean they are the wrong size (6). (A) and (C)
are not indicated.
3. What size shoe does Jana take?
(A) 5.5
- --------- .. --- -

In Eleanor's e-mail, she mentions the size is size


(B) 6 7'12.In her text, Jana writes the shoes are my size
(C) 7.5 exactly. I
(0) 8 - -- - ._------ -
4. What is indicated about Jana?
Eleanor writes that the shoes are in the office with
(A) She usually works from home.
me. Jana texts that she will drop by this afternoon
(B) She will go to Eleanor's office later.
and try them on. We can deduce she will go to
(C) She likes to wear shoes made in Italy. Eleanor's office.
(0) She prefers to buy used clothing.

5. How much will Jana most likely pay for the -- - - - ----;
, The ad gives the price as $80. In her e-mail, ,
shoes?
! Eleanor writes she will sell them at half the price I ,I
(A) $20
(B) $40 I paid for them.
------------
_ _ .

(C) $60
(0) $80

PART 7 READING COMPREHENSION 261


Passage 1/2/3

The Benefits of Functional Fitness


by Brodie Jackson, US Olympic coach
What's the point of having massive muscles if you get breathless when running for the bus, or get tired
when you carry your own children ?The trouble with many machines at the gym is that they only exercise
individual muscles, The result may be a toned, attractive appearance with big muscles, but you can't do
anything useful with those muscles,

Functional exercises, on the other hand, are those that help you handle real-life situations, These exercises
work on lots of muscle groups at once, For example, nurses, auto-mechanics, or carpenters should all try
rowing, They will find their jobs much easier as rowing builds up the muscles in the back. the shoulder, the
neck, and the arms,

Squats and lunges exercise muscles in the legs and back. Push-ups activate nearly every m~scle in the
body, especially your arm and chest muscles, The plank strengthens your core and stomach muscles, The
best exercise of all is the burpee, the ultimate full-body exercise, which combines a squat. a plank. a thrust,
and a jump; but burpees are strictly for the super-fit.

Functional exercises are adaptable to suit all ages and all levels of ability, If one is too tough for you, try
another. Start gently, build up gradually, and you'll soon be ready for a burpee,

Jake B
Hi everyone, I'm looking for advice,
I'm 42, slightly overweight, but not in
particularly bad health - I don't take
any medication apart from sleeping
tablets, The problem is I'm not that
fit. I want to get fitter, At college I
played football and baseball, but a
stressful job and four kids put an end
to that. Most evenings I collapse in
front of the television, However, I'm
determined to change all that and
to do a few of exercises that were
mentioned in Brodie Jackson's article
in the newspaper this week, Do any
of you have any suggestions?

262 READING
Main Ideas
Skim the article, web page, and online post. Then immediately answer the questions. Do not look back
at the passages.

1. Who is the article aimed at? 2. What can be inferred about Jake B?
(A) Gym owners (A) He does not play any sports.
(B) The general public (B) He enjoys watching television.
(C) Professional athletes (C) He takes no medication.
(D) Personal trainers (D) He works long hours.

Details/Inferences
Read the first question. Then scan the passages to find the answer. Only look for the answer to the
question. When you find the information you need, write the answer in the space provided. Then go
on to the next question. Answer ali the questions as quickly as you can.

1. According to the article, what are functional exercises?

2. How old is Jake B? .

3. Which exercise focuses on the arm and chest muscies?

4. What is Mr. Jackson's job? .

5. What two reasons does Jake B give for no longer playing sports? .

6. Which exercise should Jake B probably avoid? .

7. Why might health care workers benefit from rowing?

8. What sports did Jake B use to play? .

9. According to Mr. Jackson, what is the problem with most machines at the gym?

10. Who are functionai exercise suitable for?

11. Which exercise in the 20-minute Workout is NOT mentioned in the fitness article? ..

12. What should people do after completing the 20-minute Workout? .

PART 7 READING COMPREHENSION 263


Passage 4/5/6

Vanguard Office Supplies


SALE!
Stationery, printers, ink, boards, flip charts, and easels are all discounted 10%.
Shredders discounted up to 20%.

STAR BUY!
LMV Power Shredder
- Shreds up to 16 sheets of paper at a time
- Shreds staples, paperclips, credit cards, and CDs/DVDs
- U~ra-quiet performance; suitable for all shared work areas
- 100% jam-proof - powers through the toughest jobs
- Maximum runtime - 20 minutes, with a 30-minute cool down
- 6-gallon pull-out bin
- 100% safe - shredder shuts off if a hand touches the paper opening

Was $250, NOW only $200.


******************************

Last chance to buy! Sale finishes on Friday!

INVOICE

To: Vanguard Office Supplies


Target Brand Marketing 150 Adams Street
Suite 175 New York
Cornell House Invoice no. 1856
450 Lomax Street Time: 09:25
New York Date: January 31

All orders received before 10 A.M. will be delivered the same day.
Orders received after this time will be delivered the next working day.
Please make all checks payable to Vanguard Office Supplies.
If you have any inquiries about this invoice please call 555-7791 or e-mail info@vanguard.com

264 READING
Date: January 31
From: Sheryl Chambers
To: Luke Marshall
Subject: Office supplies

Hi Luke,
Did you see this ad in the local paper? It looks like a good deal. I suggest we take advantage of the promotion since
we shop at this store regularly anyway. We could use a new shredder. Ours takes ages and can only handle half the
number of sheets at a time as the model on offer. Could you pop into the store and check it out? If you're happy, then
order one. Ask them to charge it to our office account and deliver it as soon as possible.

While you're there, could you also pick up some other bargains I spotted. They are all in bins at the front of the store.
We need some color printer cartridges (get as many as they have left), four flip-chart pads, a dozen reporters' pads
for meetings, 20 ballpoint pens, and a dozen packs of their cheapest all-purpose copy paper for the printer and
photocopier. There's no harm in stocking up. The sale ends tomorrow, so you'd better go over there this afternoon or
tomorrow morning.
Thanks.
Sheryl
Office Manager

Main Ideas
Skim the advertisement, invoice, and e-mail. Then immediately answer the questions. 00 not look back at
the passages.

1. What is the purpose of Ms. Chambers' e-mail? 2. Who most likely is Mr. Marshall?
(A) To review a recent order (A) A delivery truck driver
(8) To recommend a new store (8) Ms. Chambers' assistant
(C) To promote office supplies (C) A Vanguard Office Supplies employee
(0) To request purchases be made (0) An office supplies salesperson

Details/Inferences
Read the first question. Then scan the passages to find the answer. Only look for the answer to the
question. When you find the information you need, write the answer in the space provided. Then go on
to the next question, Answer all the questions as quickly as you can.

1. In what part of the store are the ballpoint pens located? .

2. How much did the copy paper cost? .

3. What company does Mr. Marshall work for? .

4. For how long can the LMV Power Shredder operate continuously? .

5. On which day of the week was the e-mail written? .

6. What is the non-sale price of a pack of two flip charts?

7. How many sheets of paper did the old shredder shred at one time? .

8. How many color printer cartridges were available in the store when Luke got there? .....

9. What does Sheryl mean when she writes, "There's no harm in stocking up"?

10. What did Luke NOT buy that he was asked to buy? .

11. How did Luke pay for the items?

12. On which date were the items be delivered?

PART 7 READING COMPREHENSION 265


-',.-r~\rn L~~:-.
I J'" •• "- ~. I r Triple Passages
Read this e-mail, calendar, and menu. Choose the most appropriate answer: (A), (8). (C), or (D). for
each question.

To: Rahul Sharma <r.sharma@studi04.com>


From: Jack Evans <j.evans@studi04.com>
Date: October 23
Subject: Lunch next week

Hi Rahul,
Sorry, but I can't make our lunch appointment next Monday. Something important has come up.
Let's change to another day, if possible; I'm eager to discuss the upcoming Garmin project with you
and check that you're happy to work with the brief. I can do any day next week except Wednesday,
when I am in meetings all day. 12:30 or 1 P.M. works for me.
I remember you told me some time ago about your strict diet. I'm attaching the menu of the
restaurant I'm planning to book, so please do check that there's something here you can eat. If not,
we'll go someplace else. This is a great restaurant for a quick business lunch; I can recommend the
seafood - it's what I always order.

Thanks, and I am looking forward to seeing you next week.


Jack

Rahul Sharma's calendar

266 READING
The Pink Geranium
Two or Three Course Prix Fixe
Watermelonandfeta salad
Spicy chicken spring roll
Garlic-roastedasparagus

Beefbourguignon
Lemonchicken piccata
Shrimpand mussel chowder
Butternutsquashrisotto

Darkchocolatemousse
Southernpeachcobbler
Keylime pie

1. Why did Jack write the e-mail? 4. On what day will Jack and Rahul most likely
(A) To cancel an upcoming project meet?
(B) To reschedule a meeting (A) Tuesday
(C) To brief a coworker on a task (B) Wednesday
(0) To recommend a restaurant (C) Thursday
2. What time will Rahul's flight depart from (0) Friday
Milwaukee on Friday? 5. In the e-mail, the word "strict" in paragraph 2,
(A) 7:30 A.M. line 1, is closest in meaning to
(B) 10:05 A.M. (A) harsh
(C) 8:30 P.M. (B) firm
(O) 11:10p.M. (C) extreme
3. What is Jack's favorite dish on the menu? (O) limited
(A) Shrimp and mussel chowder
(B) Lemon chicken piccata
(C) Beef bourguignon
(0) Butternut squash risotto

1. (6) @ @ @ 4. (6) @ @ @
2. (6) @ @ @ 5. (6) @ @ @
3. (6) @ @ @ Score ./5

PART 7 READING COMPREHENSION 267


STEPS TO SUCCESS
This section presents an effective, step-by-step approach to use when answering questions on
Part 7 of the TOEIC@test.

The steps shown here are designed to help you achieve a higher score when you take the test. By
following these steps, you should be able to maximize your score on this part of the test.

a ~ Skim the passage to get a general idea of the content. Notice any key words and ideas.
Remember, skimming a passage means reading it quickly.

II Read the first question and look at the four answer choices. The first question often
asks about the main ideas. Answer the question if you can. If you are not sure, make
certain what information you need to look for before you look in the text.

II~ Look through the passage to find the area you need. Then slow down and scan to
find the answer. Remember, scanning a passage means reading it carefully in order
to find specific information.

Refer again to the answer choices. Eliminate any answer choices you are sure are
incorrect. Then mark your answer. If necessary, make a guess. Then go to the next
question and repeat steps 2--4.

Skimming Key Skill for Part 7


When reading for main ideas, you need to read the passage simply to get a general overview
of what it is about. This is called skimming. For example, if you look through a magazine to see
what's inside, or look at a menu, this is skimming. Do not read every word - just focus on the key I

words that contain important information. Your goal is to get an overall idea of what the text is
about. Get used to skimming texts in this way, and you will improve your TOEIC score .
.,

Scanning Key Skill for Part 7


When reading for details, you need to read the passage quickly to find the specific information
you need. This is called scanning. For example, if you look through a dictionary to find a word, or
check what time your favorite program is on TV,this is scanning. Do not read every word - just
focus on the information that you are looking for. Your goal is to find specific details. Get used to
scanning texts in this way, and you will improve your TOEIC score.

268 READING
PART 7 STEPS TO SUCCESS
Practice 1
I!mD Quickly skim this note.

From: Installation Manager, Sentinel Systems


-Iti John,
I have to leave now to ':j0 to Beta Corp. Mr. -Itolden ju.st called to say the intru.der
alarm has ':j0ne off and won't stop. -Ite thinks it's a false alarm, and ju.st wants
me to reset the system. So, ""1 sorry bu.t I won't be here for the meetln':j with Tim
C10ldmanthis afternoon. Can you. please e><plainand ':jive my apolo':jies?

NB,Try to make su.re he orders the T)(2.0 system. It's the best intru.der alarm we
have. Tell him we can install it ne><tweek, and we can install a smoke alarm (the old
5mokeC1u.ardl model we still have in stock) at half price if he wants.

Thanks,
Chris

I!mD Read Question 1. Make sure you understand what information is required. Then read the
answer choices. It you know the answer, mark your answer and go to the next question.

Im!D Quickly look through the passage to find the area you need. Then slow down and read
more carefully. Look for key words that can help you.

IfJiJD Look again at the answer choices and eliminate any you can. Then mark your answer. If
necessary, make a guess. Then repeat steps 2-4 for the remaining questions.
1. What is the main purpose of this note? 3. What does Mr. Holden want Chris to do?
(A) To reschedule a meeting with a client (A) Check if the SmokeGuard1 model is in
(8) To apologize for being unable to attend a stock
meeting (8) Install a new smoke alarm
(C) To place an order for an intruder alarm (C) Reset the intruder alarm
(0) To check prices for various kinds of (0) Reduce the price by 50 percent
alarms
4. Who most likely is Tim Goldman?
2. What kind of company is Sentinel Systems? (A) A competitor
(A) A security company (8) A coworker
(8) An investment bank (C) A neighbor
(C) A management training company (0) A client
(0) A computer manufacturer

1. @ @ @ @ 2. @ @ @ @ 3. @ @ @ @ 4. @ @ @ @

PART 7 READING COMPREHENSION 269


PART 7 STEPS TO SUCCESS Practice 2
EmD Quickly skim this letter.

Dear Ms. Hernandez:

Thank you for your letter dated August 27 asking for information about the International Hotel
Workers Association. Please find enclosed some information about our organization.

The International Hotel Workers Association's goal is to protect the rights of hotel workers all over
the world. Anyone working in the hotel industry, from part-time cleaners to hotel managers, is
able to join. We fight for better pay and working conditions for our members. We also support the
rights of our members in disputes ,,'ith employers. As a member, you will receive a newsletter every
month and be able to attend conferences and other events at no charge. Regular Membership costs
$250 per year. For an extra premium of $75 per month, we also offer a comprehensive Health Care
plan that also includes dental and vision coverage.

Please complete the form at the back of the brochure if you \vish to join. We look forward to
welcoming you into our association.

Best regards,
Mary ftolscell1
Membership Team, IHWA

•• Read Question 1. Make sure you understand what information is required. Then read the
answer choices. If you know the answer, mark your answer and go to the next question.

~ Quickly look through the passage to find the area you need. Then slow down and read
more carefully. Look for key words that can help you.

mm Look again at the answer choices and eliminate any you can. Then mark your answer. If
necessary, make a guess. Then repeat steps 2-4 for the remaining questions.

1. What is the purpose of this letter? 3. Which of the following is NOT included in
(A) To give details about an organization Regular Membership?
(B) To reply to a job applicant (A) A monthly newsletter
(C) To advertise health insurance (B) Admission to conferences
(0) To ask for a donation (C) Support in employment disputes
(0) Health insurance
2. Who does the organization aim to help?
(A) All senior hotel managers 4. Who is Mary Holstein?
(B) Mostly part-time hotel workers (A) A senior employee for a large hotel
(C) Any employees in the hotel industry (B) An administrator for a union
(0) Full-time workers who have a dispute (C) An insurance sales advisor
(0) A medical doctor

1. 0 @ @ @ 2. 0 @ @ @ 3. 0 @ @ @ 4. 0 @ @ @

270 READING
PART 7 STEPS TO SUCCESS
Practice 3
IimD Quickly skim this notice.

Attention Staff!
Next Tuesday is International Volunteer Day. Once again, we will be taking part by using our
skills to help the local community. This year we will be going to The Birches Elementary School
to install a new bathroom and paint two classrooms. We're leaving from the front entrance at
9:00 A.M., so don't be late. Nobody's late on my watch! We'll go in the two new trucks that
have our sign, E.H. Barley & Sons, on the side. Plumbers and electricians will be directed by
Rodney, and those painting will be with me. It should take a full day, but if we finish early, you'll
be free to go home! Remember, this is great publicity for us. A reporter from The Newton Times
will be coming along in the afternoon to report on progress and take some photos, so keep
smiling!

Sergei Petrov
Project Manager
E.H. Barley & Sons

EiiD Read Question 1. Make sure you understand what information is required. Then read the
answer choices. If you know the answer, mark your answer and go to the next question.

&) Quickly look through the passage to find the general area you need. Then slow down and
read more carefully. Look for key words that can help you.

I!DiD Look again at the answer choices and eliminate any you can. Then mark your answer. If
necessary, make a guess. Then repeat steps 2-4 for the remaining questions.
1. Why was this notice written? 3. What does Mr. Petrov mean when he writes,
(A) To tell staff about an event "Nobody's late on my watch!"?
(B) To ask for volunteers (A) He always checks the time carefully.
(C) To report on a local school (B) Punctuality is essential while he is in
(D) To advertise a job opening charge.
2. What are the transportation arrangements? (C) Everyone has arrived on time.
(A) They will use work vehicles. (D) His watch is always accurate.
(B) They should use their own transportation. 4. What kind of organization is E.H. Barley &
(C) Everyone should take public Sons?
transportation. (A) A teaching agency
(D) Taxis will be provided. (B) A building company
(C) A vehicle manufacturer
(D) A media consultancy

1. eEl @ @ @ 2. eEl @ @ @ 3. eEl @ @ @ 4. eEl @ @ @

PART 7 READING COMPREHENSION 271


PART 7 STEPS TO SUCCESS Practice 4
EmD Quickly skim this document.

NKP DELIVERY SERVICE - PRICES


Please note these prices are effective from Jan. 2

Delivery times Prices starting from Guaranteed on time Additional information

Standard 1-3 days, all national $3 No Maximum weight 10


Delivery locations pounds

Priority Overnight, most $14 Yes Includes $200


Delivery national locations compensation for loss
or damage

Global 3-6 days, most $35 No Maximum weight 30


Express locations worldwide pounds
Global 2-3 days, any $50 Yes Free online
Priority location worldwide confirmation of delivery

Prices based on size, weight, and distance. All packages must be sent in approved NKP packaging, otherwise
surcharge applies. Surcharge for non-standard package sizes, CDs and DVDs. Sunday/holiday delivery fee: $12.
Proof of delivery: $2.S0.lnsurance against loss or damage: from $3.75.
We regret we cannot carry dangerous and/or prohibited items. Please check our website (or details.

I!mD Read Question 1. Make sure you understand what information is required. Then read the
answer choices. If you know Ihe answer, mark your answer and go 10Ihe next queslion.

mm Quickly look through the passage to find the general area you need. Then slow down and
read more carefully. Look for key words that can help you.

IliiiD Look again at the answer choices and eliminate any you can. Then mark your answer. If
necessary, make a guess. Then repeat steps 2-4 for the remaining questions.
1. Where might you find this document? 3. How quickly could a package weighing
(A) In an instruction manual 25 pounds reach another country?
(8) In a leisure magazine (A) Overnight
(C) In a restaurant menu (8) Two days
(D) In a customer brochure (C) Three days
(D) More than three days
2. What is indicated about the Priority Delivery
4. Which of the following could NOT be sent by
service?
Standard Delivery?
(A) Some deliveries are not guaranteed.
(A) Items such as CDs and DVDs
(8) A surcharge is usually required.
(8) Items not wrapped in NKP packaging
(C) It is not available in some areas.
(C) Items costing more than $200
(D) It is an international service.
(D) Items weighing more than ten pounds

1. @ @ @ @ 2. @ @ @ @ 3. @ @ @ @ 4. @ @ @ @

272 READING
PART 7 STEPS TO SUCCESS
Practice 5
~ Quickly skim this article.

Do you ever worry that the meetings you attend are a waste of time? ---[lJ--- Research published
today by Sa reo International suggests up to two-thirds of all meetings are badly planned, poorly
managed and achieve nothing. Sarco sent questionnaires to 5,000 managers across a wide range
of industries. ---[21--- Of the 80 percent that replied, the average time spent in meetings amounted
to nine hours a week. For companies employing ten managers, that's around 4,500 hours a year. Yet
most respondents complained that meetings started behind schedule, failed to meet their objectives
and lasted too long. ---[3J--- In fact, the most effective length of time for a meeting was found to
be 30 minutes. Better training on how to run meetings properly seems to be the key. "Meetings that
are well prepared, and have an agenda, are more likely to succeed," says lead researcher Asha Sri.
As a result of the research, Sarco is planning to offer a one-week "Management Efficiency" course,
at a cost of $3,000, to companies that want to eliminate this problem. ---[4J---
Eric Knowles, Business Today

ED Read Question 1. Make sure you understand what information is required. Then read the
answer choices. If you know the answer, mark your answer and go to the next question.

mmD Quickly look through the passage to find the general area you need. Then slow down and
read more carefully. Look for key words that can help you.

I!imD Look again at the answer choices and eliminate any you can. Then mark your answer. If
necessary, make a guess. Then repeat steps 2-4 for the remaining questions.

1. Which of the following headings would be 3. Which of the following complaints is NOT
most suitable for this article? reported in the survey?
(A) Managers at Sarco Discuss Complaints (A) Meetings are boring.
(B) Successful Meeting at Sarco International (B) Meetings begin late.
(C) Study Finds Meetings of Little Value (C) Meetings do not achieve their goals.
(D) The Nine Principles of Effective Meetings (D) Meetings last too long.

2. How many managers replied to the survey? 4. In which position marked [1], [2], [3], and [4J
(A) 3,000 does the following sentence best belong?
(B) 4,000 "Well, you are not alone!"
(C) 4,500 (A) [1 J
(D) 5,000 (B) [2]
(C) [3]
(D) [4J

1. (£) @ @ @ 2. (£) @ @ @ 3. (£) @ @ @ 4. (£) @ @ @

PART 7 READING COMPREHENSION 273


Strategy Review and Tips

Strategy Review Reading Test - Part 7

Remember, in the test. ..

Always skim the passage(s) first. Don't worry about words you don't know. Keep reading. You
should read quickly to get a general idea of the content.

For each question, make sure you know what information you need to find. Then scan the passage
to find the answer you are looking for.

Eliminate any answers that you are sure are wrong.

Remember that for double or triple passages you sometimes need to refer to multiple texts to find
the answer.

Do not spend too long on one passage. Be aware of the time. If you are not sure of an answer,
make a guess and move on.

_ Here is some advice that people taking the TOf/C test have found useful for this part.
Choose the tips you like, and try to use them.

274 READING
Review Test
Directions: Read the texts. You will notice that each text is followed by several questions. For each
question, decide which of the four answer choices: (A), (8), (C), or (D), best answers the question. Then
mark your answer.

Questions 1-3 refer to the following letter.

Mr. Pierre Dubois


71 Lincoln Avenue
Wilmington, Delaware 19346

February 25

Dear Mr. Dubois:


I would like to thank you for taking the time to write to me about your latest dining experience at
Jim's Diner, Delaware. When people write to us, it is usually to make a complaint. Therefore, your
letter expressing satisfaction was a nice change.
I am delighted you found the food "excellent" and that the service was "superb." I will certainly
pass on your comments to the cook and the restaurant staff. Sandra Antonelli was your waitress that
evening. I will recommend Sandra for our "Make a Difference" award. This is a monthly bonus of
$200, given to one member of staff from any of our restaurants who has made a special contribution to
our business.
Thank you once again for your kind comments. We look forward to welcoming you again at Jim's
Diner in the near future. Please present this letter to any member of our staff for a 20% discount in
recognition of your valuable feedback.

Yours sincerely,
Mrs. Chloe Mi"elli
Manager
Jim's Diner, Delaware

1. Why did Mr. Dubois write to Jim's Diner? 3. What does Mr. Dubois receive as a result of his
(A) To complain about the poor service letter?
(8) To ask for a discount (A) A $200 cash prize
(C) To comment on a recent visit (8) A discount on his next visit
(D) To inquire about an award (C) Free entry into a competition
(D) A monthly bonus
2. How does Mrs. Minelli feel about the letter?
(A) Pleasantly surprised
(8) Rather disappointed
(C) A little upset
(D) Very curious

11.0 @ @ @ 2. 0 @ @ @ 3. 0 @ @ @ I
PART 7 READING COMPREHENSION 275
Questions 4-5 refer to the following e-mail.

o E-mail

From: I roblangley@oneworldnel.com
To: I annedarcy@vstinternel.co.us
Re: I Friday night

Dear Anne,

Sorry but I won't be able to make dinner on Friday night after all. I know I promised when you
asked me last week, but something has come up at work. Basically, my boss has just resigned
(actually, I think he was fired) and they need me to cover his job for a few weeks. There is a meeting
in Los Angeles on Friday that I have to attend. I'm flying out tomorrow morning. I'm really sorry. I
hope the evening is a success.

Let's catch up when I get back.

Rob

4. Why is Rob unable to attend the dinner on 5. The word "fired" in line 3 is closest in
Friday night? meaning to
(A) He is worried about losing his job. (A) engaged
(8) He will be on a business trip. (8) cleared
(C) He has to catch a plane. (C) promoted
(D) He is going on vacation. (D) dismissed

14. @ @ @ @ 5.@@@@1

276 READING
Questions 6-7 refer to the following text message chain.

Nigel Adams July 12, 10:34 A.M.


Hey Vicky. I'm in Granger's now. Is there anything we
need?

Vicky Skelton Ju~ 12, 10:36 A.M.


We could do with some more milk and coffee. Get one
of the big 2 kilo bags.

Nigel Adams July 12, 10:38 A.M.


You mean the red ones?

Vicky Skelton Ju~ 12, 10:39 A.M.


Yes. And get some de-caffeinated, too. Usa wants to try
it. Green bag, I think.

Nigel Adams July 12, 1041 A.M.


Got It. Anything else?

Vicky Skelton Juiy 12, 10:41 A.M.


No .... just hurry up. Jill Baker from Capcom will be here
at 111

Nigel Adams Ju~ 12, 10:42 A.M.


Ah! Forgot about that. Leaving now. Should be there
before she arrives.

Vicky Skelton Ju~ 12, 10:43 A.M.


You'll be lucky.

Nigel Adams July 12, 10:49 A.M.


Just hit bad traffic. If I'm late, give her a brochure and
tell her I'm on my way.

Vicky Skelton July 12, 10:55A.M.


She's here already. I've put her in the conference room.
Will tell her you'll be 10m.

6. At 10:43 A.M., what does Ms. Skelton mean 7. What will Mr. Adams most likely do next?
when she writes, "You'll be lucky"? (A) Pay for some groceries.
(A) Mr. Adams is usually very fortunate. (B) Speak with Ms. Baker.
(B) She expects the trip will go well. (C) Go to the conference room.
(C) She doubts Mr. Adams will be on time. (D) Continue his journey.
(D) Ms. Baker is a valuable client.

16. (6) @ @ @ 7. (6) @ @ @

PART 7 READING COMPREHENSION 277


Questions 8-10 refer to the following notice.

Leadership Seminar
Dr. Jack Dawkins

Dale & Time: October 10, 10:00 A.M. -12:30 P.M.

Location: The Marlin Cross Building, New Jackson University

Dr. Jack Dmvkins received his Ph.D from Cambridge University. He is currently Professor of
Management at Cambridge International Business School and a Senior Member of St. Matthews
College. Dr. Dawkins teaches on MBA and executive education programs. He has also taught in Japan
and Korea, as well as in France. He has twice received the CIBS Outstanding Teacher of the Year Award.
He is also C.E.O. of Electra International, a highly successful management consultancy company. He
designs and delivers a large number of corporate training programs for top international companies.

Session Content: This seminar covers the following areas:

- The seven principles of effective leadership

• Team and learn working - keeping it positive

- Management skills in the workplace

- A good manager's checklist

For information on 11010to book YOllr place. call Iall Bertram at tlze Department of Management, Extension 4342.

a. What is the purpose of this notice? 10. Which of the following does Dr. Dawkins
(A) To encourage people to enter a NOT do?
competition (A) Run a company
(B) To promote a training program (B) Teach management skills
(C) To recognize a lecturer's achievements (C) Design courses for companies
(D) To announce an event (D) Write books on management

9. Where will Dr. Dawkins' talk take place?


(A) At SI. Matthews College
(B) At Cambridge International Business
School
(C) At New Jackson University
(D) At the Department of Management

la.@ @ @ @ 9. @ @ @ @ 10. @ @ @ @ I

278 READING
Questions 11-14 refer to the following memo.

MEMO
To: All Sales Managers

From: Amanda Steel, Resources Manager

Date: March 11

Re: expenses

I have just finished analyzing the company expense reports for last month. ---[1]--- Currently, it
seems sales representatives are spending far too much on sales trips away from the office. We
are 30% over our budget already. ---(2)--- I would therefore like all sales managers to please tell
their staff to reduce their expenses significantly. From now on, overnight stays in hotels will only be
allowed for destinations over 200 km from the company. There should be no first class train travel,
even for senior managers. ---[3]--- I will also need receipts for all expenses, not just expenses
over 20 Euros as is presently the case. These measures are necessary to help us avoid cutting
expense budgets. However, if savings are not made within the next three months, budgets will
have to be cut. ---[4]---

Thank you for passing this news on to your sales teams.

11. Why was this memo written? 13. Which of the following does Ms. Steel NOT
(A To announce cuts in expense budgets mention?
(8) To ask sales staff to reduce their (A) Travel by air
expenses (8) Trips by rail
(C) To report the company's latest losses (C) Receipts
(D) To encourage fewer sales trips (D) Accommodations

12. What does Ms. Steel intend to do in three 14. In which position marked [1], [2], [3], and (4)
months' time? does the following sentence best belong?
(A) Cut spending by sales staff "I'm afraid there will be no way to avoid it."
(8) Reduce the number of sales staff (A) [1]
(C) Review expense budgets (8) [2]
(D) Announce cuts of up to 30 percent (C) [3]
(D) [4J

11. @ @ @ @ 12. @ @ @ @ 13. @ @ @ @ 14. @ @ @ @

PART 7 READING COMPREHENSION 279


Questions 15-18 refer to the following article.

TEN TOP TIPS IN BUSINESS (continued)


Tip No.5

[n business. creating a strong relationship with your Step Two is to prepare a plan of how you can help
clients is important. It allows you to understand your clients. Set yourself achievable goals. and
their needs and get the best possible results. review the plan regularly. It is important to assess
your progress. to find out how well you are doing.
Step Olle is to be informed. Take some time to
Always do what you say you are going to do. Keep
learn about your clients' interests and ambitions.
any deadlines you promise your client.
Find out about them and their business. Look at
their annual reports. and browse for information These are the fundamental principles of successful
on their business activities on the internet. Most of relationships in business. Anyone who can
all. talk to your clients about what they do and how maintain relationships effectively will succeed.
you can help them achieve their goals. Can you?

15. What is this article about? 17. Which of the following is NOT mentioned in
(A) International business Step Two?
(B) Managing staff (A) Never break your promises
(C) Business relationships (B) Establish realistic targets
(D) Successful companies (C) Keep within your budget
(D) Measure your progress
16. In the first step. which of the following
principles is most important? 18. The word "fundamental" in paragraph 4,
(A) Find out about the client's company line 1, is closest in meaning to
(B) Discover the client's goals (A) basic
(C) Research the client's needs (B) amazing
(D) Communicate with the client (C) truthful
(D) compulsory

15. @ @ @ @ 16. @ @ @ @ 17. @ @ @ @ 18. @ @ @ @

280 READING
Questions 19-21refer to the following form.

Kilvers Translation Services


Forrest Place
Perth
WA 6001

Customer Satisfaction Form


Thank you for coming to us for your language translation needs. We would like to know how
satisfied you are with the service you received. Please take a few moments to complete this form
by circling your answers. Your help will allow us to improve our performance. All answers are
confidential.

not satisfied at all highly satisfied


Value for Money 2 3 Q} 5

Quality of Work

Time taken 1 @
2 3

3
4

4
en
5

Staff
cD 3 4 5

Overall 2
0 4 5

Would you recommend Kilvers Translation Services to others? Yes ~


Reasons:
As always, I was very pleased with the final translation done by Ivan, bu.t this ti",e
it took over one week "'ore than e)(pected. This cau.sed proble",s with the co"'pany in
MoscowI was sendin'j the contract to. I al",ost lost the order'

Comments/Suggestions:
Perhaps you.shou.ldtrain you.r reception staff to be "'ore polite and friendly when
dealin'j with telephone in4u.iries.I called several ti",es to ask abou.t the delay, bu.t the
",an I spoke to (c;re'jory?) was u.npleasant on every occasion.

19. Which of the following can be inferred from 20. The word "unpleasant'. in paragraph 3, line
this form? 3, is closest in meaning to
(A) The customer has used this company (A) inaccessible
before. (B) disagreeable
(B) The company mostly translates (C) unhelpful
documents into Russian. (0) impractical
(C) The translators are all very experienced.
21. Who most likely is Gregory?
(0) The company sends a feedback form to
(A) The owner
all its customers.
(B A translator
(C) A receptionist
(0) A customer

119.0 @ @ @ 20.0 @ @ @ 21.0 @ @ @ I

PART 7 READING COMPREHENSION 281


Questions 22-23 refer to the following fax.

FAX Smith, Parson & Shaw


Investment & Wealth Management Specialists

Date: Wednesday May 27


Time: 13:23 EDT
To: Diane Sotton, Office Direct
From: Sofia Barkley
Pages: 3
Subject: Order Ref: 76327
Message: Dear Diane,
Please find following our order for stationery supplies for next month. It's bigger than
normal, as we have our summer training program starting next week. ---[1]--- I hope
you can get all these items to us by Friday, June 5, as courses will start the following
Monday. The Legal-size Office Pads (NV324) are probably the most important, but
please let me know if there are any items you can't supply. You'll notice we need five
different colors of Presentation Binders (FD165) - this is essential, so please don't
send all the same color as you did last year! I noticed the black toner cartridges
(TG436) are a good price at $35, so I ordered ten. ---[2]---
Since the order totals over a thousand dollars, I understand there is a 20% discount.
As usual, we'll pay within 30 days of receiving your invoice. ---[3]---
If you need any further information, or want to arrange for special delivery methods,
just give me a call on Extension 437. ---[4J---
Regards,

Sofia Barkley
Office Manager

22. When does Ms. Barkley want the items to be 23. In which position marked [1]. [2]. [3]. and [4]
delivered? does the following sentence best belong?
(A) As soon as possible "1 hope you have enough."
(B) On Monday, June 8 (A) [1 J
(C) Before the end of the following week (B) [2]
(D) Within 30 days (C) [3]
(D) [4]

122'(£) @ @ @ 23. (£) @ @ @

282 READING
Questions 24-25 refer to the following memo.

MEMO
Aspect Corporation
From: SumikoSato
To: All Employees
Date: Tuesday,March 4
Subject: Your support, please!

Hi, everyone!

As some of you may know, I am taking part in the Boston Marathon next month. This year,/ am
running in support of the American Stroke Association. Pleasebe generous and sponsor me if you can.
Last year I raised $3457 for Cancer Relief,and this year I want to do even better. I started training
two months ago.It's hard! Hopefully, I will finish in less than four hours (my record is three hours and
two minutes). Anyway,don't feel like you haveto give me anything (I know many of you havealready
contributed to the company's official charity. UNICEF). but whatever you do give will go to a
good cause.

Thank you very much,

5umiko

24. What does Ms. Sato mean when she writes, 25. What can be inferred about Ms. Sato?
"I war:Jlto do even better"? (A) She likes running marathons.
(A) She wants to run faster. (B) She has run many marathons.
(B) She wants to raise more money. (C) She has run the Boston Marathon
(C) She wants to work harder. before.
(0) She wants to achieve more. (0) She runs in several marathons every
year.

1 24.(£; @ @ @ 25. (£; @ @ @

PART 7 READING COMPREHENSION 283


Questions 26-29 refer to the following schedule.

United Health Medical Centers


Summer Intensive
Course Objectives
To help medical staff at UHMC develop their management and leadership competencies.
Participants will explore strategies to organize, motivate, and lead staff effectively.

Location
The course will take place at the Chatfield Institute, a residential management training venue
10 miles outside New York.

Schedule
Sunday 6
A.M.
10:30 Arrival and check-in
11 :45 Course Introduction

P.M.
2:00-3:30 Key Seminar: "Challenge, Opportunity, and Success" Dylan McKenzie
4:00 Team Building (see course folders for more details)
7:00 Welcome Reception, the Belvoir Room

Monday 7
A.M.
9:00-10:30 Key Seminar: "Performance Management in Hospitals" Linda Chang
10:30-12:00 Key Seminar: "Healthcare Strategy and People Development" Maurice Hugo

1:15-5:00 Visit to Meegan Pharmaceuticals, Inc. (bus leaves from main entrance)

Tuesday 8

9:30-12:00 Visit to MicroSystem Wellness Center (meet at side entrance)

1:00 Team project work


7:00 Farewell reception, the Rosen Suite

Wednesday 9

8:30-1 :30 Team Presentations. These will be assessed by Vice President Gail Winderson,
of Human Resources.

2:00 Check-out. Bus leaves at 2:30 P.M.

284 READING
26. Who is this training course for? 28. The word "competencies" in paragraph 1,
(A) University teachers line 1, is closest in meaning to
(B) Healthcare professionals (A) preferences
(C) Local government employees (B) gifts
(D) Military personnel (C) skills

27. What further information can participants find (D) choices


in their course folders? 29. Where will participants stay during the
(A) Details of team-building activities course?
(B) Biographies of the key speakers (A) In New York
(C) Guidance on the team project work (B) At the training center
(D) Departure times for bus journeys (C) In their own homes
(D) With their team members

26.0 @ @ @ 27. 0 @ @ @ 28. 0 @ @ @ 29. 0 @ @ @

PART 7 READING COMPREHENSION 285


Questions 30-34 refer to the following notice and e-mail.

Sakura Building 1
Notice to All Occupants
July 20
Fire Alarm Test
In order to meet our insurance responsibilities, every year we have to test the fire alarm systems in the building.
We have therefore scheduled Monday, August 17 for a full fire alarm test. Please note that this affects all
occupants of ollices on everv floor of the building.
The fire alarm is a continuously ringing bell. When you hear the fire alarm, you must leave the building
immediately.
General Evacuation Procedures
Stop your work.

Stay calm.
Close, but do not lock, your office door and window.
Use the nearest sare stairs (do not use the elevator) and make your way to the nearest exit.
Go to the Emergency Assembly Area (EAA) in the parking lot.

When everyone is out of the building, please wait until the Fire Officer has announced it is safe to re-enter.
Thank you for your cooperation.
Building Management. Sakura Buildings Group

E-mail
o
From: I kwalliam@presoneat.com
To: I management@sakurabuildingsgroup.com
Re: I Yesterday's fire alarm test

Dear Sirs,

I really must complain about yesterday's fire alarm test. Although you gave us the date, you did not give
us a time. As we had no idea when the alarm would sound, many workers in my office felt unable to make
calls to clients, hold meetings, etc., because they didn't want to be interrupted. In the end, the alarm
sounded at 5:20 P.M., when some staff had already gone home. What's more, although we evacuated our
office, 1 noticed people from several other companies continued working as usual.

Can you please make sure that the next fire alarm is during normal office hours, and that everyone follows
the correct procedures? It would also be good to know what time you intend to sound the alarm.

Sincerely,

KenWaliiam

CEO" Presoneat Assurance Co., Ltd

286 READtNG
30. Who is Ken Walliam? 33. When did Mr. Walliam send his e-mail?
(A) A manager in the Sakura Buildings Group (A) June 17
(B) An employee in the Building Management (B) July 20
office (C) August 18
(C) The head of a company with offices in the (0) October 4
building
34. In the notice, the word "scheduled" in
(0) The Fire Officer who carried out the test
paragraph 1, line 2, is closest in meaning to
31. What problem with the fire alarm test does (A) planned
Mr. Walliam NOT mention? (B) designed
(A) The fire alarm was ignored by some (C) imagined
people. (0) expected
(B) The test took place too late in the day.
(C) Not everyone could hear the fire alarm.
(0) They had no idea when the test would
take place.

32. According to the notice, what should office


workers do when the alarm goes off?
(A) Check that their coworkers are safe
(B) Leave as quickly as possible
(C) Lock all doors and windows
(0) Wait for instructions from the Fire Officer

30.0 @ @ @
31.0 @ @ @
32.0 @ @ @
33.0 @ @ @
34.0 @ @ @

PART 7 READING COMPREHENSION 287


Questions 35-39 refer to the following advertisement and e-mail.

Maintenance Supervisor
Location: Atlanta Airport, Atlanta, USA

Pay: $50,000 - $55,000 per year


Employee Type: Full-time

Job Type: Management

Experience: 3 to 5 years

Deadline: September 30
Contact: Vivien McAllister vm@atlantaairporthr.com

Description
US Union Airlines is currently looking for a Maintenance Supervisor at our Atlanta facility. You will be
responsible for the supervision and management of the day-to-day operations within the Maintenance
Department.
Requirements
3-5 years experience in aircraft maintenance

High School Diploma or equivalent


Current FAA Certificate, held for at least 5 years

General business knowledge


The successful applicant will also have basic computer skills, including knowledge of common
computer software programs. You should be knowledgeable in all aspects of the Maintenance Manual
requirements. You should also have excellent leadership and communication qualities, with strong time
management. Applicants with at least one year's supervisory experience will be preferred.

o E-mail

From: I Ryan Lyton<rly23@usinternet.com>


To: I vm@atlantaairporthr.com
Re: I Maintenance Supervisor
[Attachment - R.Lyton n3sume.doc)

Dear Ms, McAllister:

I would like to apply for the position of Maintenance Supervisor at US Union Airlines. Please find
attached my resume for your attention. As you will see, I achieved my FAA Certificate in 2001, and
have eight years' experience in aircraft maintenance. Although I have never been a manager, I strongly
believe I have the right skills to fulfill the job well. I am used to working with computers and am familiar
with all the latest software.

If you want to get in touch, the best way to reach me is on my cell phone - '-749-023-9083, or you can
e-mail me at this address.

I look forward to hearing from you.

Best regards,

Ryan Lylon

288 READING
35. According to the advertisement, what should 38. How does Mr. Lyton prefer to be contacted?
people interested in the position do? (A) Bye-mail
(A) Call Ms. McAllister (B) By phone
(B) Apply before the end of September (C) By letter
(C) Send a resume and photograph (0) Byfax
(0) Complete the application form
39. In the job advertisement, the word
36. What is the successful applicant required "common" in the final paragraph, line 1,
to do? is closest in meaning to
(A) Supervise staff (A) frequent
(B) Run training sessions (B) popular
(C) Travel frequently (C) famous
(0) Work flexible hours (0) advanced
37. Why might Mr. Lyton's application not be
successful?
(A) His computer skills are poor.
(B) He has insufficient experience.
(C) He has only just qualified.
(0) He has no previous management
experience.

35.0 @ @ @
36.0 @ @ @
37.0 @ @ @
38.0 @ @ @
39.0 @ @ @

PART 7 READING COMPREHENSION 289


Questions 40-44 refer to the following notice, e-mail and text message.

Mel Eversfield in association with

Rialto Recordings
presents
for one night only

the giants of the rock music world

THE SHAKING WILLOWS


playing all their classic hits
Musical guests include the Grammy award winner for solo artiste

Carmen Carrero

Saturday September 5
Weslern Slale Hall, Atlanla
8:00 P.M.

Tickets available from: www.rialtorecordillgs.com

Date: Tuesday August 26


To: All sales staff < sales@copydatacom>
From: Emily Lane <e.lane@copydata.com>
Subject: Concert tickets

Dear all,
I have two lickels that I cannot use for the Shaking Willows concert on September 5. I bought the
tickets earlier this year as a fifth wedding anniversary present for my husband, but I just found out
that my knee operalion has been scheduled for lhe day before lhe concert! They say I will be in
the hospital for 48 hours. Anyway, the tickets cost me $60 each, but I am willing to sell them at a
discount. All offers considered.
Sincereiy,
Emily
Sales Administrator

290 READING
Rebecca Wong 1147
Hey Emily. Re your e-mail - if those tickets are still
available, I'll give you half the price you paid for them.
How's that? BTW, sorry about your knee!

40. What type of music will be heard at the 43. How much in total does Rebecca offer for the
concert? tickets?
(A) Rock (A) $30
(B) Classical (B) $50
(C) Operatic (C) $60
(0) Country and western (0) $120
41. What is NOT indicated about Emily? 44. What will most likely happen on Sunday
(A) She works in the same department as September 6?
Rebecca. (A) The Shaking Willows will perform again.
(B) Her knee operation has been (B) Rebecca will go to the Western State
rescheduled. Hall.
(C) She got married five years ago. (C) Both women will meet Mel Eversfield.
(0) She made a purchase from Rialto (0) Ms. Lane will be discharged from the
Recordings. hospital.
42. On what day of the week will Emily's
operation take place?
(A) Wednesday
(B) Thursday
(C) Friday
(0) Saturday

40.(6) @ @ @
41.(6) @ @ @
42.(6) @ @ @
43.(6) @ @ @
44.(6) @ @ @

PART 7 READING COMPREHENSION 291


Questions 45-49 refer to the following Web page, memo, and e-mail.

TEAM-BUILDING DAYS
Energize your staff!
Improve workplace dynamics!
Challenge your people!

HAVE FUN!
The average office is a very stressful environment - and where there's
stress, there's conflict. Colleagues fallout, productivity drops, breakdowns in
communication affect performance, office politics get out of control ...

If this describes your office, then we have the answer. If you want people to
perform better at work, they need to work well together, and feel better about
themselves and their job. Here at Buzz Events we can help. We know that
removing people from their everyday environment, giving them challenging
tasks, and encouraging them to collaborate in non-threatening, friendly
surroundings, can result in better relationships between individuals and
heightened productivity and effectiveness. So, while your staff de-stress
and have fun, you see results!

Try one of our popular team-building activities:

Dragon boat racing


Dragon boats are like big canoes. Your employees have to work together to
win. One of our staff keeps time to make sure the team is in order by banging
on a big drum! The winning team is the one that is able to operate in unison.

Raft building and racing


Teams construct their own raft - and hopefully make it to the other side of our
lake in the ensuing race. It's either that or sink and swim! This activity is huge
fun. It builds confidence and encourages creativity and teamwork.

Scavenger hunt
Teams have a list of items that they must search for and locate by solving
clues. Teams have to think creatively and outside of the box and really work
their gray matter in this game. The only way to succeed is to think and work as
a team.

Pontoon challenge
The challenge in this activity is to use the materials we supply to build a
structure that is big enough, strong enough, and buoyant enough, to allow
every member of the team to cross to the middle of the lake.

292 READING
MEMO
From: Office Manager
To: All staff
Subject: Team-building day
Dear all,

The management team has decided to introduce something new this year.Instead of our usual party in August
we thought it would be fun for you to spend a day with your coworkers outdoors and get to know them better
by working together on some fun tasks.So,on Wednesday,July 19 we'll be going on a team-building exercise.
There are various options (see the link below).We will all do the same activity as a group, but you can choose
the activity. Readthe information, and let me know your preference by Friday, June 8.The activity with the most
number of votes wins!

Sincerely.
Stephanie Chase
Office Manager

From: Ryan Morton <info@buzzevents.net>


To: Stephanie Chase <s_chase@technospheric.com>
Subject: Team-building event

Dear Ms. Chase,


This is a courtesy e-mail to confirm that we are looking forward to seeing your group, consisting of
twenty-six people, next Wednesday. We will be preparing the boats this week. You will have two teams of
thirteen, plus two or three of our own staff will join you, including our expert drummer and timekeeper. If
numbers change, I would appreciate it if you would notify me as soon as possible. Please arrive at least
45 minutes before the scheduled start time of the activity.
Thank you.
Ryan Morton
Client Relationship Co-ordinator

45. What is the main purpose of Ms. Chase's 48. In which month did Ryan Morton write his
e-mail? e-mail?
(A) To publicize the results of a recent survey (A) June
(B) To announce a change of date for a party (B) July
(C) To attach an important company (C) August
document (0) September
(0) To ask staff for their input on a matter
49. Which team-building activity will
46. Which benefit of team-building is NOT Technospheric staff take part in?
mentioned in the Web page? (A) Dragon boat racing
(A) Employee stress levels will be reduced. (B) Raft building and racing
(B) Productivity in the workplace will go up. (C) Scavenger hunt
(C) Interpersonal relationships will improve. (D) Pontoon challenge
(D) Staff will demonstrate more loyalty.
45.0 @ @ @
47. On the Web page, the phrase "fall out" in
46.0 @ @ @
paragraph 1, line 2, is closest in meaning to
(A) Have disagreements 47.0 @ @ @
(B) Become sick 48.0 @ @ @
(C) Change places 49.0 @ @ @
(0) Leave work early
PART 7 READING COMPREHENSION 293
Questions 50-54 refer to the following notice, Web page, and e-mail.

CALL FOR PAPERS

The annual Philosophical Society conference will be held this October at Harvard
University. The topic of the conference will be German Thinkers of the 19th Century.
Leading experts in this field have been invited to speak and we are pleased to announce
that Professor Hans Hoffman has agreed to give the keynote speech on Hegel and
Idealism.

The co-chairs at this conference will be Professor Jeremy Roberts and Dr. Shelley Jung.

You are invited to submit papers on topics that you feel would be of interest to participants.
Current post-graduate students and post-doctoral researchers are particularly encouraged
to consider submitting details of their research. Papers should have a reading time of
approximately 40 minutes. Papers themselves should not include the author's name or
affiliation, as the review process will be blind; this information should be submitted in a
separate document. Please visit the Registration and Submissions page for details.

Submissions are due by July 31.

Georgeville University Department of History


Marlon Phillips, Professor of History of Thought

Professor Phillips specializes in European philosophy including Manifesto for Revolution. which considers
and political thinkers. He came to Georgeville the philosophy and ideas of the 19th-century German
University in 2009 after having taught for twenty years thinker Karl Marx. and Authority and Totalitarianism.
at Cambridge University in England. He gained his which considers the ideas of the 20th-century Gem,an
B.A. from Columbia University in 1980. and his Ph.D philosopher Hannah Arendt. He is currently working
from Harvard University in 1985. He has published on a book about classical German philosophy.
articles on a wide range of topics. including socialism. Professor Phillips teaches the undergraduate
rationalism. idealism, and authoritarianism. His first courses German Philosophy. Russian Philosophy. and
book. Leibniz and Rationalism. is regarded as the world The History of Ideas. He is currently supervising a
authority on the 17th-century German philosopher number of Ph.D. students in these fields.
and mathematician Leibniz. His second book. The
First Russian Marxist. about the] 9th-century Russian Professor Phillips will be on sabbatical leave from
thinker and revolutionary. Plekhanov. won the Royal September to December of this year. when he will be
Society "Book of the Year" award in 2005. In recent conducting research at the University of New South
years he has written other award-winning books, Wales in Sydney.Australia.

294 READING
To: Professor Marlon Phillips <m.phillips@georgeville.edu>
From: Gregory Cecil <g_cecil@harvard.edu>
Date: March 25
Subject: Philosophical Society conference, October 25-27

Dear Professor Phillips,

On behalf of the Philosophy Department of the University of Harvard, I am writing to invite you to speak at
the annual Philosophical Society Conference, to be held at Harvard from October 25-27. The title of the
conference is German Thinkers of the 19th Century. You are one of the leading experts in this field, hence
our invitation. As you know, the conference attracts not only academies, but also undergraduate and post-
graduate students, as well as members of the general public with an interest in philosophy. The conference.
organizing committee feels that your book on this period will be of interest to the audience, and we would like
you to talk about the subject for about forty minutes. If you are happy to accept our invitation we can decide
on a specific time and date for your talk, and can discuss the content in more detail.

I do hope you will accept this invitation, and I look forward to hearing from you.
Sincerely,
Gregory Cecil
Administrator, Department of Philosophy

50. What is true about Professor Phillips? 53. Which book does Gregory Cecil most likely
(A) He has met Gregory Cecil before. want Professor Phillips to talk about at the
(B) He is an Australian citizen. conference?
(C) He speaks German fiuently. (A) Leibniz and Rationalism
(0) He previously lived in Britain. (B) The First Russian Marxist
(C) Manifesto for Revolution
51. What can NOT be inferred about the October
(0) Authority and Totalitarianism
conference?
(A) Members of the general public may 54. Why will Professor Phillips probably turn
attend. down the invitation?
(B) Post-graduate students may talk about (A) He will be out of the country.
their research. (B) The subject is not his specialty.
(C) The registration date for attendees is (C) He will be working on a book.
July 31. (0) He has teaching commitments.
(0) The conference takes place every year.

52. What is indicated about review process?


(A) Reviewers will not know who wrote the 50.(£) @ @ @
papers. 51.(£) @ @ @
(B) Reviewers must not show the papers to
52.(£) @ @ @
anyone.
(C) Reviewers will not publish their opinions.
53.(£) @ @ @
(0) Reviewers will not be experts on the 54.(£) @ @ @
subject. Score ............./54

[---

,....----_. __
ExTRA PRACTICE ONLINE I
._----------------'
Go to www.pass-the-toeic-test.com for advice on how to improve your reading comprehension
.1
skills.
~

PART7 READING COMPREHENSION 295


Reading Test
This Reading Test covers Parts 5, 6, and 7 of the TOEIC" test. Allow 75 minutes to complete this test.
Mark all your answers on the separate Reading Test Answer Sheet provided on page 325.

Part 5
Directions
Read each sentence. You will notice that there is a word or phrase missing. Study the four answer
choices and select the one answer: (A), (8), (C), or (0), that best completes the sentence. Then mark
your answer on the Answer Sheet.

101. There was no elevator in the office building, 104. Mr. Saito has been away on a business trip
which was very for visitors to the ......
Monday.
higher floors. (A) on
(A) inconclusive (8) since
(8) inefficient (C) during
(C) incomplete (0) by
(0) inconvenient
105. Following the oil leakage, the company had
102. We do not have money in the cash to pay a of $12,000 for polluting the
register. environment.
(A) most (A) fare
(8) much (8) tariff
(C) many (C) fine
(0) almost (0) tax

103. Nobody answered the door when I rang the 106. Gary enjoyed so much at the party
bell, so they have left already. that he did not get home until very late.
(A) must (A) he
(8) can (8) him
(C) did (C) his
(0) would (0) himseif

296 READING
107. Please can you all. .....attention during 111. I.... . to work this month because I need
the safety demonstration? the exercise.
(A) give (A) walk
(8) get (8) am walking
(C) pay (C) had walked
(0) hold (0) was walking

108. All of our used cars are sold with a six month 112. I do not think there is chance that
.........................
on parts and labor. interest rates will rise this month.
(A) contract (A) no
(8) assurance (8) some
(C) guarantee (C) any
(0) promise (0) none

109. When the workers arrived the 113. The report made several comments
factory gates, they found the company had about the bonuses given to senior staff.
gone out of business overnight. (A) critic
(A) to (8) critical
(8) on (C) critically
(C)at (0) criticism
(0) by
114. The chairman promised to the
110. I am sorry, but smoking in this area is possibility of raising salaries for workers in
.....................
.....forbidden. the cafeteria .
(A) strictly (A) show up
(8) steadily (8) look into
(C) severely (C) keep on
(0) sternly (0) come across

lij.,.JlII••• :I#I@I::g"#41C1:+

READING TEST 297


115. We should be on time for the movie ..... 119 we sat at the back, we could hear
the traffic is worse than normal. the presenter very clearly.
(A) if (A) However
(B) so (B) Despite
(C) because (C) Although
(D) unless (D) Even

116. Celine first has to take a management training 120. One thing we always.. .. on here at
course ._ by her company if she wants Clarkson Engineering is punctuality.
to qualify for promotion. (A) demand
(A) provided (B) insist
(B) proclaimed (C) order
(C) proceeded (D) require
(D) produced
121. Expanding our business interests into Asia
117. Is Miguel coming with us to seems to be a very decision, given
Colorado on Friday? the recent growth in the region.
(A) positively (A) reasonable
(B) definitely (B) reason
(C) undeniably (C) reasonably
(D) certainly (D) reasoning

118. Because of the failure of the marketing 122. There is nothing... .. to a company than
campaign, Frederick decided he had no loyal, hard-working staff .
.............................
but to resign from the company. (A) as important
(A) alternative (B) so important
(B) substitute (C) important
(C) decision (D) more important
(D) opportunity

298 READING
123. The coffee shop my brother works for 127. Both of these candidates are very strong, so
..........................
him to bring home three bags of deciding them will be difficult.
coffee a week free of charge. (A) among
(A) lets (B) between
(B) allows (C) inside
(C) agrees (0) within
(0) consents
128. What is our to overcome this
124. The coach leaves twenty minutes downturn in profits and beat the recession?
from Platform C. (A) strategy
(A) during (B) strategic
(B) until (C) strategically
(C) in (0) strategical
(0) before
129. Can you ..........
me another cup of coffee,
125. The winner of the first prize in the design please?
competition .. . finally been (A) take
announced. (B) allow
(A) was (C) bring
(B) did (0) carry
(C) has
130. Everyone at the welcome party was
(0) is
dressed and the atmosphere was relaxed.
126. If there is anything else I can do to be of (A) actually
assistance, please do noL to ask. (B) casually
(A) hesitate (C) careiessly
(B) hesitated (0) needlessly
(C) hesitant
(0) hesitation

••I:I:I@I*'.:mct:
lij.l.i~.,

READING TEST 299


Part 6
Directions: Read each text. You will notice that there are four blanks. These are places where a word,
phrase, or sentence is missing. For each blank, study the four answer choices and select the one
answer: (A), (B), (C), or (D), that best completes the text. Then mark your answer on the Answer Sheet.

Questions 131-134 refer to the following memo.

MEMO
To: All Sales Dept. staff
From: Computer Dept.
Date: Nov. 23
Re: Network maintenance

An important update to the company's computer network is ~..~.~ for the afternoon of Friday,
November 28. Work will commence at 2:00 P.M. We expect it to continue until some point after
midnight.

For 1.~? this period, it will not be possible to access the Sales and BOOkings system. E-mail
will also be unavailable, along with the company website. All public workstations will be out
of ~.~~ Access to e-mail and electronic data via cell phones or tablet PCs will also be
unavailable .

..~..~.~ For example, you may wish to transfer files to a notebook, to use during the afternoon.
If you need help, do not hesitate to ask.
We apologize for any inconvenience.

Computer Department

131. (A) declared 134. (A) Please make suitable arrangements in


(B) scheduled advance.
(C) aimed (B) We all need to try harder to make
(D) maintained progress.
(C) Teams should work closely together.
132. (A) most
(D) Work in the afternoon can be more
(B) mostly
productive.
(C) most of
(D) the most

133. (A) action


(B) debt
(C) print
(D) office

300 READING

You might also like